NBDE Part 2 Sample Questions and Answer 2020

(use Dr. fun for second day, master day 2, unicorn)

 

Caterpillar corrected saba

EL magnifico

speedy snail

corona RQ

Dr. Fun july 2020
mamba mentality

DO strawberry next and dr xylitol

 

 

 

ENDO:

Intraoral dental sinus: Parulis (made up of granulation tissues, AKA gumboil from chronic abscess sinus/fistula) no tx other than ENDO, from chronic periapical abcess from necrotic tooth, if it doesn’t drain then I and D

Most likely cause pulp necrosis after trauma to the tooth: Pulp hyperemia
perforation prognosis worst: below osseous crest then furcation

Doing endo which part of MX Incisors perforate (root curved disto-lingually)? Mesial

Doing endo which part of Mx PM perforate? Mesial

doing endo, which part of Mn M1 perforate: D of mesial canal then M of distal

contraindication in endo: recent MI, uncontrolled DM

contraindication of pulec/pulpo in: leukemia.
after RCT marked reduction in size in bone: 6 mt-1 year to make

Dentist restoring tooth with amalgam after RCT should place amalgam 3mm inside

Blood pigments, pulp hyperemia, internal resorption, cervical external resorption after bleaching: red tinge, pulp necrosis: grey color, pulp calcification: yellow color

Percussion: presence of inflammation in PDL or not

Palpation: spread of inflammation to periodontium from PDL or not

EPT– responsiveness of nerves

Thermal test (hot & cold)- pulp vitality. Hot (irrev), cold (rev)

How do you differentiate between an endo/perio lesion: EPT
Differential diagnosis of acute periodontal abcess & acute periradicular abscess? EPT
in Periapical abscess: gram-negative obligate anaerobic tx: penv/amoxi +metro, clinda
reccurent abcess: e.fecalis + facultative anaerobes (amoxi+metro-strinct anaerobes), clinda

Patient with tooth that has sensitivity that lingers with thermal test, sinus tract, and positive to percussion, what does the patient have? Irreversible pulpitis with chronic periapical abcess
Dx lingering pain to cold and sensitivity to percussion: irreversible pulpitis with Periapical periodontitis
Dx not responsive to cold, not to percussion, and palpation is tender: necrotic pulp and chronic apical periodontitis.

Your going to do RCT for upper lateral with Periapical abscess for which of the following possibility you have to take informed consent from the patient? (benefits, risks, prognosis)

1 Broken instrument

2 long term prognosis

3 perforation

4 ledge formation

Ans all of the above

 

Which teeth do you perform pulp evaluation on?

tooth only

tooth and neighboring tooth

tooth, neighboring teeth, contralateral tooth

tooth, neighboring teeth, opposing tooth
Ans C

 

Endodontic pain is characterized by all except:

Dull, aching pain

Sharp, shooting pain

Throbbing Pain

Electrical Pain

Ans D (galvanic shock)

 

Prolonged, unstimulated night pain suggests which of the following conditions of the pulp? A. Pulp necrosis

Mild hyperemia

Reversible pulpitis

No specific condition

Ans A

 

The most important principal governing the location and outline of the lingual or occlusal opening into the pulp chamber is:

1 preserving tooth structure

direct access along straight lines

complete removal of roof and pulp chamber

removal of all caries and defective restorations materials

ans 2

 

Apical detector is use for all of these except:

length of the canal

reduce uses of periapical rx for lengthening checking

detects accessory canals

dilacerations
Ans C and D

 

a Periapical lesion was discovered 1.5 years after an uncondensed, single cone root canal filling was placed in max central incisor. Two year after careful Periapical curettage, lesion is larger than it was before surgery. Likely cause?

1.Systemic involvement

2.Inadequate curettage

3.Failure of resect apex

4.Undiagnosed perio lesion

5.Leakage from poorly filled canal

Ans 5

 

Best to test pulp status for newly erupted tooth with open apex

1 hot test

2 cold test

3 Electrical pulp test

4 percussion

Ans 4

 

Which can differentiate between acute apical abscess and acute periodontal abscess?

Palpation

Rx, Anesthesic test

Percussion

EPT

Ans 5

 

Avulse permanent tooth immediately comes to the clinic what is your first response?

A implant it

B take radiography

Ans: implant it, Then take x-ray (# 1 determining factor of prognosis of avulsed tooth is time, the sooner the better, within 20 min is the best chance of survival of PDL)

 

Which of the following is the diameter, in millimeters, of a 21 mm long, #35 K-file at D16?

A- 0.35

B- 0.41

C- 0.67

D- 0.74

And C (assume 2% taper unless stated otherwise 0.35+(2%x16)=0.67 mm) =D1 +(2%x16)

During endo treatment dentist made a ledge what is the reason?

1-changing large file often

2-using small file

3-curved root

ans 1.

 

During the preparation of a Class II cavity, which of the following permanent teeth pulp

horns will be the most subject to accidental exposure?

Distofacial of a maxillary first molar

Distofacial of a mandibular first molar

C Facial of a mandibular first premolar

Lingual of a mandibular first premolar

Ans C

Which root of the following is LEAST likely to form ledge on it

1 long

2 curved

3 short

4 narrow

Ans 3 (short, broad no calcification least likely)

 

for which factor is least likely to refer endo case

Dilacerations

Calcification

Inability to obtain anesthesia

Mesial inclination of molar

ans: D

which of the following nerve fibers of pulp are responsible for thermal conduction?

a) A beta

b) A delta

c) A beta and A delta nerve fibers

d) A alpha and A delta nerve fibers

e) C fibers& A delta

Ans E

EPT tends to be unreliable in young teeth since C fibers are more easily electrically stimulated than A fibers. In Young teeth A fibers appear later than C fibers.

A) Both statements are TRUE.

B) Both statements are FALSE.

C) The first statement is TRUE, the second is FALSE.

D) The first statement is FALSE, the second is TRUE.

Ans D

Most important detail that would most affect the outcome of a fractured tooth

a) 48 hour delay of treatment

b) fracture being far away from apex

c) larger than normal pocket

d) infection

ans B

talking about fractures here, not avulsions (where time and medium of storage are most important) fracture line will determine tx and also prognosis of the tooth. (simple fracture in enamel and dentine only vs complicated fracture of Cr and root)

worse prognosis of tx as we go coronally towards alv crest. better prognosis if its near apex.

 

Which of the following anatomic structures CANNOT be seen on periapical

radiographs?

A- Mental foramen

B- Hamular process

C- Mandibular foramen

D- Anterior nasal spine

E- Intermaxillary (median palatine) suture

Ans C (also can’t see coronoid notch)

 

A patient experienced a blow to the mouth. Radiographs show a horizontal mid-root fracture of a maxillary central incisor. The tooth is NOT mobile and NOT symptomatic; however, it does not respond to pulp testing. No radiographic lesion is present. The best treatment is to

1) institute root canal treatment to include both segments of the tooth.

2) extract the coronal segment and surgically remove the apical segment.

3) surgically remove the apical segment and reverse fill the coronal segment.

4) render no treatment at this time and periodically recheck clinically and radiographically.

Ans 4

 

RQ Which one has best prognosis of RCT

1) internal resorption with close to perforation

2) external resorption

3) Gutta percha expanding beyond apex

4) incompletely debride canal

Ans 3 (s.s) internal resorp would be better but if close to perforation, its weak structure and could fracture.

 

Each of the following is basic objectives in the cleaning and shaping of a root canal EXCEPT one. Which one is this EXCEPTION?

A) Removal of the infected soft and hard tissue

B) Give disinfecting irrigants access to the apical canal space

C) Create space for the delivery of medicaments and subsequent obturation

D) Removal of the smear layer by opening the dentinal tubules

E) Retain the integrity of the radicular structures

Ans D

 

The pain characteristic that may provide information regarding its etiology while taking pain history is:

A) Intensity

B) Quality

C) Onset

D) Temporal pattern

E) Alleviating factors
ans C

 

Each of the following statements about chronic hyperplastic pulpitis is true EXCEPT one. Which one is this EXCEPTION?

A) Most commonly present in Young adults and children

B) Prognosis for the pulp is unfavorable

C) Less current than normal is required to elicit a response by Electric pulp tester

D) Tooth may respond to thermal test when ethyl chloride is used

E) Polypoid tissue is less sensitive than normal pulp tissue and more sensitive than gingival tissue
ans C

 

Once the ledge is bypassed with an instrument, further instrumentation should be done w:

A) Circumferential filing

B) Clockwise three-quarter turns

C) Clockwise quarter turns

D) Twiddling like motion

E) Paint on brush strokes

ans A

 

if you have pain, what would be the hardest to anesthetize?

Irreversible pulpitis and maxillary

Irreversible pulpitis and mandibular

Necrotic pulp and maxillary

Necrotic pulp and mandibular

Ans B

 

Succuess of indirect pulp capping depends on

1 age of the patient

2 exposure location

3 coronal seal

4 thickness of CAOH

Ans 3

 

additional treatment beside RCT for a sinus tract

1 surgical excision

2 nothing

3 antibiotics

Ans 2

 

You re implant Avulsed tooth with non rigid fixation how long you will leave it

1 5-7 days

2 10-14 days

3 18-21 days

4 28-30 days

Ans 2

 

Most Perforations on max lateral root during RCT is

1 mesial

2 distal

3 buccal

4 lingual

Ans 1

 

71-Which of the following is NOT characteristic of external root resorption

Appears irregular in X-ray

Could be accompanied with ortho movement

Can happen in re-implanted teeth

Asymptomatic

none of the above

Ans 5 non of the above b/c 1 and 2 are true for sure maybe 4 (s.s)

 

replacement resorption is characterized by:

pain

Apical fibrosis

Acute inflammation

ankylosis

ans 4, also dull metallic sound, infra-occlusion

Which of the following can cause submerge

1 ankylosis

2 internal root resorption

3 missing teeth

Ans 1

 

Best prognosis of broken file at the apical third

1 vital with no preapical lesion

2 Vital with preapical lesion

3 non vital with preapical lesion

4 non vital with no preapical lesion

Ans 1

Broken file in apical third while instrumenting w/o radioL what to do

1 extract

2 apeocectomy

3 obturate and fellow up

Ans 3

PDL mostly affected by

1 avulsion and intrusion

2 extrusion

3 lateral laxation

4 Concussion

Ans 1 ( thanx god there was no avulsion option )

Pulp Necrosis occurs in avulsion, intrusion** 90% of time.
intrusion in primary teeth: no tx, let spontaneous re-eruption. Unless bud underneath then EXT
in permanent teeth w/ immature apex: no tx, spontaneous reeruption (if more than 6mm repos)
intrusion in permanent teeth with mature apex: more than 3mm orthodontic extrusion or immediate surgical reposition should be considered always do these with splint (2 week)

Q10: Which of the following is the best treatment for a traumatically intruded primary tooth which in not impinging on the permanent tooth bud?
A- Extraction
B- Allow tooth to spontaneously re-erupt
C- Reposition tooth orthodontically
D- Reposition tooth surgically
ANS: B

little girl has ALL, radiolucency in furcation of primary M2 what to do?
EXT
Pulp
Pulpec
Ans. 1 ok so in child w/ leukemia pulpotomy/pulpectomy  is contraindicated, but for permanent teeth no if it can be completed within one day. Ideally do EXT 2 weeks prior to cancer tx. adult contraindication to ENDO: leukemia RCT is NOT a contraindication, recent MI and uncontrolled DM, COPD, renal failre, CHF etc (asa4)

Cracked tooth with no pulpal involvement, treatment?

RCT

Extra-coronal (crown)

Reduce occlusion

Ans: B (but if option there to see if pulpal involvement or not do that first)

 

craze line vs. cracks differentiated by: transillumination.
crack: tooth is cracked (M-D), the light will be blocked, allowing only a segment of the tooth structure to light up, or use X-ray at 90 degree and 45 degree
if only has a craze line, the entire tooth structure will light up (only in enamel)
VRF: CBCT best way to dx or x-ray, then probe defect after long standing, most common in Mx PM1, to diagnose “J”

PERIODONTICS

Most common donor site of free gingival graft: Palate

Main reason for failure of FGG: lack of blood supply, Infection (2nd most)

2 most critical parameter for prognosis perio: CAL** and mobility

Bacteria responsible for collagenase activity: P. gingivalis

Most common perio: chronic (in black males)

Most common pattern of osseous defect in chronic periodontitis: Horizontal/crater

Most common teeth lost by perio problem: MX M1

Best allograft: FDBA cadaver

Chronic Periodontitis: Most common in Black males, people older than 65

Post operative sensitivity after perio surgery decreased by: doing-plaque control

Initial re-evaluation after SRP: 4-8 weeks. Maintenance: 3 months
Localized vertical bone loss on molars seen on which periodontitis:
localized aggressive localized aggressive periodontitis bacteria? AA (8-12 healthy, doesn’t correlate to plaque)
Generalized aggressive periodontitis :
P.intermedia and E. corrodens (episodic rapid 12-25)

Chronic perio: P.gingivalis, T. forsythia
ANUG bacteria:
fusobacterium(before), P. Intermedia t. denticola
antibiotics LAP: doxyciline or amoxicilin+metro

Antibiotic for ANUG (only if systemic symptoms): amoxicilin+metro or Tetracyclin
most important indication of prognosis of periodontal tooth: CAL

Most important indication of perio stability after maintenance: plaque

Indication for success of tx: BOP

Drug induced Gingival Hyperplasia: Dilantin (Anti-convulsant), #1, Phenytoin (Anti-convulsant), Cyclosporin, Nifedipine, Verapamil (Calcium Channel Blocker), Diltiazem (Calcium Channel Blocker)

Desquamative gingivitis: liquen planus, pemphigoid, pemphigous, chronic ulcerative stomatitis, lupus, linear IgA, erythema multiform

Biological width: 2mm, JE + CT (0.97+1.07)

Biological width of implant: 3-4mm

GTR: Coronal movement of PDL * complete regeneration

After performing root planning, how does the new attachment form, or after periodontal surgery? Long Junctional epithelium (reparative)

In health CEJ to alveolar crest: 2-3mm

CAL (CEJ to base of pocket)= PDD+rececion

Gracey scalers: area specific, cuts on one edge, semi cirucular in cross sx, 60 degree bevel, 13/14 for Distal
implant epithelium: hemidesmosom, osteointegration

Hyperplastic tuberosity method: wedging technique (distal wedge)

FGG: nutrients from recipient but healing by proli from epith adjacent from donor (indication widen keratinized giniva, apical to free gingival margin, make it more thick)

Calculus: 12 days to form

Pedicle seconds, adhesion attachment mins, plaque colonization 12-24hrs (first facultative gram +ive aerobs, then filaments until day 7 then after 1 week gram – anaerobes) plaque first formed in interprox

During Sr/p: dentin, cementum and calculus removal

disadvantage of cancellous autograft: lack of strength

problems with chemotaxis in neutrophil: can lead to Aggressive perio

Attached gingiva thickest in lateral maxilla (most) and least in mandibular first premolar and canine(least): true

 

Intermedia is shown to increase in all of the following except

A pregnancy induced gingival enlargement

B ANUG

C Down syndrome

D diabetes

E orthodontic therapy

Ans E

Least inflammation associated with?

Chronic inflammation

Atrophic gingiva

Aggressive periodontitis

Desquamative ginigvitis

Ans D

 

What is the primary indication for splinting?

Mobile teeth and pt’s discomfort and better control of occlusion if front are mobile

Trauma from occlusion
c. improve OH

Prevent unopposed tooth from migrating

Prevent migrating after diasthema closure
Ans A (advantage of external splint over internal splint: conservation of tooth structure)
Indications for splinting: (1) mobility of teeth that is increasing or that impairs patient comfort, (2) migration of teeth (3) prosthetics where multiple abutments are necessary.

 

What not related with the splinting multiple mobile teeth?

To comfort pt & facilitating chewing

Prevent more bone loss

To keep the graft in place (stent)
Ans B

 

Compared to a full thickness flap, a partial thickness(split-thickness) flap will

increase the loss of marginal bone.

reduce infraosseous defects.

provide improved surgical access.

increase the amount of attached gingiva.

reduce healing time

 

Which of the following is NOT a characteristic of a modified Widman Flap procedure?

A- Submarginal incision

B- Replaced flap

C- Inverse bevel incision

D- Flap margin placement at the osseous crest

Ans D

 

You rise a full mucoperiosteal flap to instrument in the pocket, after reposition of the flap where resorption occurs more?

Radicular bone (not crest but bone covering the tooth, 1mm)

Interdental crest

Apical to the sulcus

 

most common sign of tooth trauma from occlusion:

tooth sensitivity

b) tooth mobility

ANS B=fremitus(X-ray: PDL Widening, lamina dura thickening, angular bone loss, root resorp, hypercementosis) for fremitus can also selectively grind down . then we see sensitivity, widened PDL, thicken lamina dura

 

Most common to cause mobility?

Trauma

Advanced perio

Periapical pathology

 

Patient came to visit your office, complain that she shows too much gum when smiles. How to fix the patient’s problem?

Le fort 1

Gingival flap

Gingivectomy

Crown lengthening

Ans: D because most cases due to passive eruption. if its due to vertical hyperplasia of maxilla A, if do to drug induced gingival hyperplasia gingivectomy if excessive attached gingiva. if not enough attached gingiva and bone is ok, do apically position flap. if due to delayed/altered passive eruption and too much bone esthetic Cr lengthening (if alv crest to CEJ is less than 3mm, cut bone so apically positioned flap + osteotomy)

posterior tooth has a large carious lesion extending subgingivally. Which of the following is the best initial treatment?
A- Endodontic therapy
B- Crown lengthening surgery
C- Caries excavation
D- Crown fabricatio n
Ans C

Biological width after Cr lengthening: could be osteotomy + APF or osteotomy+ gingivec
A general rule of thumb for a crown preparation is that you should have 3 mm between the margin of the preparation and the crest of bone to ensure adequate crown length. You will need 2/1.5 mm more for the prep to remain on sound tooth structure. The amount of tooth necessary required to expose for proper retention is 5 mm.

Which of these have the best response to root amputation?
Mx M1
which have best response to root hemisection/premolarization? Mx M1
older people loose which tooth first to perio: MX M1

Which of the following conditions indicates that a periodontal, rather than an endodontic problem, exists?

Acute pain to percussion with no swelling

Pain to lateral percussion with a wide sulcular pocket

A deep narrow sulcular pocket to the apex with exudate (endo-perio/root fracture)

Pain to palpation of the buccal mucosa near the tooth apex

Ans B

Which of the following has worst perio prognosis?

class 2 mobility

deep class 2 furcation

deep probing with suppuration

ans C (class III mobility, sever CAL,class III mobility)

 

Which one is LEAST cause for extraction

1 grad 2 mobility

2 deep endo-perio caused by perio

3 non restorable tooth

4 vertical root fracture

Ans 1

 

which cell doesn’t present in all stages of chronic periodontitis?

A- PMNL

B-eosiophhils

C- plasma cells

D- lymphocytes

Ans B

Healing after a gingivectomy?

Primary Intention

Secondary Intention

Tertiary Intention

Ans 2

 

what is common between chronic periodontitis and generalized aggressive periodontitis?

1-the teeth that are involved

2-the rate of progression of the disease

3-the response to the local factors
ans 1 (local fact like Ab, GAD can use systemic Ab for chronic we give SDD doxy subclinical)

 

aggressive periodontitis is characterized by

1 loss of bone around 1st molars

2 loss of bone around canines

3 horizontal bone loss

4 associated with plaque

Ans 1

 

Most common tooth lost by perio problems

1 max premolars

2 max molars

3 man premolar

4 man anteriors

Ans 2

 

Improperly drained puss from perio pocket leads to

Abscess

Cyst

Granuloma

Ans A (Acute abscess, Chronic Granuloma, can go back and forth from there, from there can go to acute osteomyelitis or cyst)

 

What will interfere more with a lateral position flap?

Frenum attachment

Amount of keratinized gingiva

Amount of nonkeratinized gingiva

Amount of mucogingival junction

Ans A (gingival thickness and width of attached gingiva at donor)

A posterior tooth has a large carious lesion extending subgingivally. Which of the following is the best initial treatment?

A- Endodontic therapy

B- Crown lengthening surgery

C- Caries excavation

D- Crown fabrication

Ans C

 

Periodontal destruction:

1) is a continuous process.

2) occurs in episodic and intermittent manner

3) shows periods of destruction and quiescence

4) destructive periods are marked by increased non motile gram +ive microbes

5) quiescence period is marked by gram +ive non motile microbes.

A 1 only

B 2 3 5

C 2 4 5

D 3 4 5

Ans B.

 

Of following conditions, inflammatory gingival enlargement is least characteristic of:

Desquamative gingivitis

pregnancy

hereditary fibramatosis

Phenytoin induced hyperplasia
5. Leukemia

ans 3

 

95- Appearance of an osseous crater in a bitewing x-ray will show:

More bone than what is actually lost

Less bone than what is actually lost

Like a cervical burnout

No difference

Ans b

 

 

 

Which systemic disease does not especially predispose to perio?

Cyclic neutropenia

Trisomy 21

Leukemia

sarcoidosis

Diabetes mellitus

AIDs

HIV

osteoporsis

Ans: (LAD1, LAD2, AIDS, papillon lefevre syndrome, chidiak hegashi LAP diabetes, smoking, are all risk factor chronic (risk factors which predisposes you) and risk indicator (which is not causally associated but can potentiate higher risk) and HIV/osteoporosis/socioeconomic level are risk indicator… not a risk factor donno tbh….

 

26-When you take Graft from a Pig it considers as

1 xenograft

2 allograft

3 autograft

Ans 1

 

64- Which interleukin is related to bone destruction?

1 IL1

2 IL2

3 IL6

4 IL10

Ans 1

 

Red edematous gingiva, no rete peg, marginal gingivitis , what initial treatment

1 oral hygiene and plaque control

2 scaling and root planning

Ans 1

 

Which one has least benefit from antibiotic besides mechanical debridement

1 LAP

2 GAP

3 Chronic periodontitis

4 periodontal abscess

Ans 3

Best prognosis for GTR b/w these options:
1. Hemiseptum
2. 3 walled defect (trough)
3. Furcation II class
4. Tunnel furcation
ans. 2 (trough) then 3 (class II furcation) shallow 3 wall better with osteectomy

Best prognosis of GTR: 1 wall, 2 wall 3 wall 4 wall (then II furcation)
worst prognosis of GTR: 1 wall

Which of the following is true regarding treatment and prognosis?
A. Perio lesion from pulp lesion has better prognosis
B. Endo lesion from perio lesion has better prognosis
C. Perio treatment before endo has better prognosis
D. Perio surgery has better prognosis for perio-endo lesions
ans A.

Recent studies prove periodontal disease related with

Cardiovascular disease

Cancer

Hypertension

Ans A

For big osseous defect in mandible which graft is the best

1 Allograft

2 Autograft

3 Dried frozen

Ans 2

Which of the following red complex bacteria

1 Treponema denticola

2 s mutans

3 A.actinomycetcomitans

4 provetella intermedia

Ans 1 (Treponema denticola, p. gingivalis, Tarentella forsyth)

 

you did scaling and root planning to pt after that he went back with perfect plaque control but still have bleeding on probing and 6 mm pocket depth what is the next step

1 another scaling and root planning

2 no thing

3 open flap surgery

4 oral hygiene instructions

Ans 3

 

In periodontitis, the loss of bone always correlates to

A depth of periodontal pocket

B severity of ulceration of pocket wall

C presence or absence of exudate

D all

E none

Ans. E, none

 

Which part of the cutting edge of the curret should be adapted to the line angle of the tooth?

A- Lower third

B- Middle third

C- Upper third

D- Entire cutting edge

ANS:A

 

Which of the following is the best initial treatment for a patient with localized aggressive periodontitis?

A- Dental prophylaxis plus subgingival irrigation

B- Dental prophylaxis

C- Scaling and root planing

D- Scaling and root planing plus systemic antibiotics

Ans: D (initial just srp + antib as adjunt. Check in re-eval phase 4-8 weeks after),
prophy given if pt already taking penicillin, give another class like clinda ** for sure on ADA

what is the primary etiologic factor in generalized aggressive perio (GAP):
a. altered lymphocytes
b. generalized subgingival calculus
c. impaired PMN
d. bacterial plaque
ans D (still plaque there containing AA in LAPand GAP is e corrodens but amount not associated with destruction)

 

Best toothbrushing technique

Stillman

Charter

Sulcular

 

In examining a maintenance patient, the dentist observes residual calculus, bleeding on probing, and probing depths less than 5 mm. The dentist should do which of the following?

A- Scaling and root planing

B- Osseous surgery

C- Continued maintenance

D- Open flap debridement

Ans A

 

The most predictable for treatment of perio disease?

Scaling & root planning
2. The modified Widman flap
3. Free gingival flap
4. Apically positioned flap
ans A

FLAPS:

Indications for electrosurgery: coagulation, hemostasis, removal of hyperplasic tissue, gingival cords, CR lengthening
Contraindications to electrosurgery
: thin attached gingiva (dishicense/fenestration), in pt with TENs unit, cardiac pace maker, insulin pump, delayed healing (diabetes, cushing)

Gingivoplasty indication: reshaping of papilla and gingiva to give form for function, ANUG

To surgically remove pocket wall:  external bevel gingivectomy and undisplaced flap (internal bevel gingivectomy) *undisplaced most used
incision of Gingivectomy: Coronal to mucogingival junction, and removes pocket coronal to pocket base (reduces pocket depth, A beveled incision is made apical to the pocket base)
indication of gingivectomy: in drug induced hyperplasia, fibrous hyperplasia, pseudopockets and supraboney pocket
contraindications to gingivectomy: infrabony pockets (defects), lack of attached keratinized gingiva, compromised esthetics. If base of pocket is located apical to osseous crest or at MG junction, do perio flap), high caries index

Deciding between full/partial thickness flap: depends on amount of keratinized gingiva (>2mm for split minimum)

 

Gingivectomy is NOT indicated when the base of the pocket is located

A- Apical to the alveolar crest

B- Below the free gingival groove

C- Coronal to the cementoenamel junction

D- Apical to the cervical convexity of the tooth crown

 

Which is most likely the major consideration prior to performing a gingivectomy?

A- Amount of attachment loss

B- Measurement probe depth

C- Level of the alveolar crest

D- Width of the attached gingiva
Ans D Width of attached gingiva will give Pocket depth and Location of the mucogingival junction

NARROWEST BAND of attached gingiva is found on FACIAL SURFACES of the mandibular canine & first premolar, and lingual surfaces adjacent to mandibular incisors & canines. Narrow attached gingival zones may also occur at the MB root of maxillary first molars (associated with prominent roots and sometimes with bony dehiscences), and at mandibular third molars.

 

The primary factor for selecting periodontal flap surgery rather than gingivectomy is

presence of gingival edema.

pocket depth.

presence of subgingival calculus.

need for access to the bony defect.
ans D

 

External bevel of gingivectomy apical to:

Mucoalveolar junction

Gingival fiber

Epi attachment

Alveolar bone

Ans 3

 

The base of the incision in the gingivectomy technique is located
A. in the alveolar mucosa.
B. at the mucogingival junction.
C. above the mucogingival junction.
D. coronal to the periodontal pocket.
E. at the level of the CEJ junction
ans: C

Gingivectomy, external bevel incision

90 degree incision facial or lingual toward the tooth

just Apical to pocket depth

Coronal to Mucogingival junction, just above

Ans: B

 

Modified Widman flap: Variation of replaced flap (not displaced). Full-thickness flap used for purpose of open flap debridement, facilitating instrumentation and regenerative procedures. Heals by primary intention and with long JE. Not intended to reduces pocket depth, but when removing pocket wall it reduces pocket depth b/c it establishes a new attachment at a more coronal level. Preserves an adequate zone of attached gingiva.
indications: shallow/moderate pockets w bases coronal to MG junction, high esthetic regions

modified widman flap is

Partial thickness flap apical to mucogingival junction

Full thickness flap apical to mucogingival j

Partial thickness flap coronal to mgj

Full thickness flap coronal to mgj

D

The modified Widman flap:

1)in addition to improving accessibility for instrumentation, removes the pocket wall, thereby reducing or eliminating the pocket

2)facilitates instrumentation but does not attempt to reduce pocket depth

3)improves accessibility and eliminates the pocket, but does the latter by apically positioning the soft tissue wall of the pocket

4)none of the above

Ans 1

With a modified Widman Flap, you mostly reduce bone if:

1-Adapt the flap margin

2-Ossesous restructuring

3-removal of infected ossesous tissue

4-removal of malignancy tissue

ans 1 (usually we do not do osseous recontouring in MWF only in WF)

 

Which of the following is NOT a characteristic of a modified Widman Flap procedure?

A- Submarginal incision

B- Replaced flap

C- Inverse bevel incision

D- Flap margin placement at the osseous crest

Ans D (this is done in widman where flap places apically, not modified widman. In MW it placed at same position)

 

The modified widman flap achieves pocket depth reduction by

a) Shrinkage only

b) Removing the pocket wall

c) Displacing the flap coronally

d) Displacing the flap apically

Ans A occurs in healing

 

AFP objective: of this type of full-thickness flap is to surgically eliminate deep pockets by positioning the flap apically while retaining the keratinized gingiva. Surgical acess, tx of infraboney pockets, root planning. Usually used with osseous surgery. Needs vertical insicions. On palatal must cut tissue. Work horse and high predictability
AFP indications: surgically eliminate mod/deep pockets, furcation, Cr lengthening.
AFP Contraindicatation: w/ root caries and where root exposure is unaesthetic.

 

Best position for apically displaced flap?

A.at the alveolar crest

B.at 2mm apical to alveolar crest

C.at 1 mm apical to alveolar crest

D.at one mm coronal to alveolar crest
ans B

 

Correction of an inadequate zone of attached gingiva on several adjacent teeth is best accomplished:

apically repositioned flap.

laterally positioned sliding flap.

double-papillar pedicle graft

coronally positioned

free gingival graft

Ans E

 

Which of the following flap designs allows the best surgical access to the apical aspect of a tooth root with the least reflection of soft tissue?

1)Envelope

2)Semilunar

3)Vertical release
ans 2

 

What has the biggest effect on the flap?

A Initial incision

B Extensive ness of reflection

C Post op oral hygiene

D Final position of the flap

Ans C

 

flap in all the mandible which structure may damage

1 mental nerve

2 facial

3 orbicularis oris muscle attachment

4 mentalis muscle attachment

Ans 1 (common in mandibular vestibuloplasty)

 

Incision made in vestibuloplasty with mucosal graft

Subperiostal

Supraperiostal

Submucosal

Supramucosal

Ans B (to increase relative height of alveolar ridge by apically positioning buccinators, mentalis and mylohyioid)

 

What is the most common form of wound healing after a periodontal flap surgery?

A- Long junctional epithelium

B- New connective tissue attachment

C- Connective tissue adhesion

D- Regeneration of the new periodontal ligament, cementum and bone

Ans A

 

Surgical flap access therapy is indicated and most beneficial when used

A- For those early to moderate defects not resolved with initial therapy

B- As the initial treatment for patients having extremely heavy subgingival calculus

C- To eliminate pocketing more rapidly so the patient can proceed with treatment

D- To improve plaque control effectiveness in patients having difficult achieving good plaque control

ANS:A

A variation of the laterally positioned flap is called:

(1)a coronally positioned flap

(2)a modified Widman flap

(3)a double papilla flap

(4)a free gingival flap

36 infrabony/vertical/angular defect is best determined by:
A. Probing measurement
B. Radiograph
C. Full flap exploration
D. other option
ans C.

Of the 4 critical zones of pockets surgery, Phase I therapy solve many of the problems on this particular zone. Which is this zone?

A zone 1 soft tissue, pocket wall

B zone 2 tooth surface

C zone 3 underlying bone

D zone 4 attached gingiva

Ans b

 

Which of the following is incorrect?

A padicle flap requires donor site to have thick and wide gingiva

B free connective tissue graft requires thick tissue at donor site

C sandwich type flap require gingival thickness at donor site

D pouch and tunnel require gingival thickness at recipient site

Ans C

 

Only undisplaced flaps can be used on palate

For a flap to be displaced apically, coronally or laterally, it must be a full thickness flap

A 1 is correct and 2 is incorrect

B 1 is incorrect and 2 is correct

C both correct

D both incorrect

Ans B

 

Crestal incisor and sulcular incisions are always Internal bevel incision

Sub marginal incision can be internal bevel or external bevel.

A) 1st correct, 2nd is incorrect

B ) 1st incorrect and 2nd is correct

C) both correct

D) both incorrect

Ans C

 

In Apically positioned full thickness flap, vertical incision may or may not be needed

In coronally displaced full thickness flap, Vertical incisions are a must

A) 1 is correct, 2 is incorrect

B 1 is incorrect, 2 is correct

C) both correct

D) both incorrect
Ans C APF can be used without vertical incision. For coronal, vertical incision are required.

 

Free gingival graft indication (FGG): widen attached gingiva apical to free gingival margin covering dishecense/fenestration, prevent further recession. Where >2mm of attached gingiva exists. FGG epithelium sloughs and dies and new ones comes from adjacent mucosa and surviving basal cell. 1 week epithelium 2 weeks tissue appears but until 10-16 weeks matures.
FGG dependent on the bed of recipient blood vessels periosteum. Fails b/s distruption of blood suppy then infection. Usually done with frenectomy.
free mucosal graft/ CT graft indication: widen attached gingiva coronal to free gingival groove to increase. Root coverage where <2mm of attached gingiva. FMG epithelium type comes from donor site (palate)

After a free gingival graft, primary source of nutrition for graft during the first 24 hours is

A primary vascular anastomosis

B residual nutrients within the graft

C diffusion of nutrients from the underlying connective tissue

D diffusion of nutrients from adjacent vessels of the periodontal ligament
ans C

 

epithelium of a FGG undergoes degeneration at the recipient site

Genetic information as to the nature of the epithelium overlying the connective tissue is contained within the graft connective tissue

A- Both statement are true

B- Both statement are false

C- The first statement is true, the second is false

D- The first statement is false, the second is true

 

Best Plastic flap surgery for root cover

1 lateral pedicle graft

2 coronal positioned flap

3 Free gingival flap

4 connective tissue flap

Ans 4-gold std. (pedicle graft/coronal if there’s enough keratinized tissue to cover, miller I and II ) coronally advanced flap is not indicated for recession >3 mm.)

 

MATERIALS:

Most strong porcelain: firing under compression
What increases with age: chroma

What decreases with age: value

What’s more important: value

How to prevent Metamerism? Look under diff light

What can’t occur when we add stain? Increase value (we cannot increase value/brightness)

Wavelength: Hue (to change hue use orange)

how to reduce value: add complementary color (purple)

Most important dimension of color, dental restorations/whiteness/darkness of teeth: value

Margin discoloration of Veneer:

Day: Amine

Week: silane

Month: microleakage and Microcrack

Green discoloration of entire porcelain: Silver, at margin: copper

Most dimentionally stable impression: Addition silicone (PVS) can stay for 1 week, has best everything but sulfure in latex can inhibit. No byproducts but Hydrogen gas if moisture

Condensation silicone: give ethanol as byproduct

Least dimentionally stable: condensation silicone and hydrocolloid

most rigid/stiff: polyether (shortest working time, fastest setting time), hydrophilic, imbibition with water (worst hydrocolloid)

Strongest resistance to tearing: polysulfide (by product is water, syneresis)

Best wettability(adhesion): hydrocolloids, polyether (least condensation silicone)

Best casting: hydrocolloid then PVS

Zoe: add water to accelerate and oil to retard

RMGI What is the advantage beside fluoride release: Anticariogenic, chemical bond

GIC: chemically adherent to tooth structure(acid base)

Color stability in resin: UDMA (less amine as TEGMA is for viscosity)

Resin initiator of polymerization in light cure: camphorquinone

Resin initiator in self cure: benzoyl peroxide

 

Patient has a premolar A2 PFM crown and its not good at esthetics because teeth are all B1 main problem in the crown?

a) value

b) chroma

c) transluceny

d) hue

 

Main difference and advantage of using GMT instead of Enamel hatchet?

bi-angled cutting surface

angle of the blade

push/pull action instead of

Ans b

 

Color family is

1 hue

2 value

3 chroma

4 pigmentation

Ans 1 (how to change hue: Add orange)

 

Color property which give strength, intensity, saturation to the color

1 Value

2 Chroma

3 hue

4 translucency

Ans 2

What property makes a substance liquid over compression?

isometric

isotropic

thixotropic

Ans C

 

What is not an advantage of rubber dam when compared to not using it:

Improved properties of materials,

shortens operative time

facilitates the use of water spray
ans. B

 

Placement of rubber dam affects color selection by?

Dehydration of tooth

Rubber dam make tooth look lighter

Rubber dam interfere with the light creating shadows

Ans A

 

Which one of the flowing require LONGEST time for acid etching

1 Zirconia (no etch)

2 Feldspathic (etch 120 sec)

3 Lithium disilicate (etch 20 sec)

4 leucite Ceramic (etch 60 sec)

Ans 2 (120 s, zirconia/aluminum don’t need b/c don’t have silica)

 

Which one is Composite monomer:
1 Methacrylate

2 Dimethacrylate

3 methyl methacrylate

Ans 2 (MMA monomer for acrylic resin, PMMA polymer for acrylic resin)

 

when pouring gypsum material into an impression, which material causes the LEAST amount of bubbles?

Polysulphide

Polyether

Silicone

Irreversible hydrocolloid

Ans 4 (best wettability, castability)

 

All of the following can do in Veneer, except?

Try in paste

Silane application on surface

Bonding agent application

Etch enamel with hydrofluoric acid

 

What should be added in Composite to increase it color stability?

A-BIS-GMA

B-UDMA

C-PMMA

D-HEMA

E-TEGDMA

Ans: Colour B, viscosity TEGMA

 

Tooth past anti sensitivity

1 Cacl2

2 KNO4 (potassium nitrate) (Naf, Snf2)

Ans 2 (lauryl sulfate for detergent, pyrophosphate for antitartar)

 

Highest coefficient of Thermal expansion

Enamel

Amalgam

Resin

Gold

Ans: resin (COTE) unfilled resin even more

 

Highest thermal conductivity: gold, then amalgam. (need bases), GIC then Resin

 

 

high modulus of elasticity, low elongation, high strength is

1 ductility

2 brittle

3 tough

4 malleable

Ans 3

 

Which of the following is responsible for creating a green discoloration in the marginal area of the metal ceramic restoration?

A- Copper

B- Silver (if whole crown)

C- Cobalt

D- Palladium

Ans A. if margin, B if whole

 

Oil or water on impression for treatment casts causes

An increase of the quality

No alteration

A decrease of the quality

Bubbles on the cast

None of the above

And D

 

Which cement is the easiest to remove after procedure? Zinc Phosphate cement/ZOE

Alginate impression in 100% humidity, what will occur? imbibition

Condensation silicone release: Ethyl alcohol

The most stable elastic impression in moisture environment?
a. Polyether

Additional silicone (PVS)

Condensation silicone

Polysulfide

ans b

 

indium other oxygenated material added to porcelain why?

Increase retention

Decrease opacity

Increase bonding

Ans. 3

 

gypsum need to high water/powder ratio

Type I

Type II

Type III

Type IV

Ans. Type I

https://files.sld.cu/protesis/files/2018/06/chapter10-rests-and-preparations.pdf

 

EDTA Function: chelating agent. Causes no harm to periapex. Dissolves inorganic

 

Impression not for fixed prosthesis

Reversible Hydrocolloid

PSV

irreversible hydrocolloid

Ans C

When doing composite, what inhibits polymerization?
A. Zinc oxide

Eugenol

Water
Ans B

 

The most difficult to remove with high speed handpiece?

Gold alloy type 3

High noble gold

Noble gold

Base metal

Ans: base metal
(ADA type IV gold: strongest of all gold (75% nobel) yield strength >340

base metal: was nonprecious metal before: < 25% noble metal content (no gold required) yield strenghth 820. Base metal has higher everything meting point, stronger etc.. compared to gold EXCEPT lower specific gravity)

OPERATORY:

Critical Enamel pH: 5 (dentin and cementum 6.2-6.7) with fl (enamel goes to 4.5)

Caries initiation: Streptococcus Mutans for initiation esp on pit and fissure number one

caries progression: Lactobacillus for progression (and smooth surface)

Root caries: s. mutans and actinomyces

First to be seen in plaque: Sanguis **

arrested carries: shiny black

chronic leathery brown dark

rampant: leathery and soft surface

increased type of caries in last years: root, b/c of gingival recessions

optimal fl-: 0.7 (but in states, 0.7 ppm-1.2ppm depending on location set by EPA)

http://www.ada.org/~/media/ADA/Files/Fluoridation_Facts.pdf?la=en

most common is 1ppm (1mg) because of temperature

class IV resin: 45 degrees, 0.2-2mm bevel, 0.2mm axial wall depth into dentine

class V: anterior GIC (GIC, eroded anterior, class V anterior, class III but low stress)

posterior class V: amalgam

class III distal canine: never resin, best is gold or min amalgam

liners: 1-25 micrometers (varnish same copalite)

zone 2,3 of dentine there’s remineralization

need to stop bleaching 7 days before resin bonds (sclerotic dentine doesnt bond)

desensitizer: GLUMMA

primer: HEMA (MMA)

bond/adhesive: is unfilled bis-GMA

class II gingival bevel for gold inlay: 40 degrees (gold inlay has slightly smaller marginal gap when compared to resin)

minimum tooth reduction in working cusp: 2.5 for amalgam, 1.5 gold,

in general Class 5 you can keep unsupported enamel in gold

most caries: Mn M1

least caries: Mn I

ditching 0.5 in amalgam observe if more like 0.6mm then replace (if amalga has reccurent carries, remove a little and put some more)

Bonding is difficult to where: Sclerotic dentin is not removed
Restoration covering the cusp: Resistance form
after big MOD patient come back with pain in drinking cold and after examination .. the tooth has pain in cold stimulus that subside quickly and bite exam was normal .. tooth is not sensitive to percussion ; diagnosis ? reversible pulpitis

Onlay: indicated for hyper/hypoeruption, when need to cover 1/3 intercuspal, primary retention: parallelism of vertical surface (axial walls). Sharp point and line angles increase onlay retention. Never shoe functional cusp always cap (complete) gold WC clearance: 1.5mm NWC: 1 mm

Inlay: better contour, better contact better everything than amalgam (make sure no undercuts)

Gold cast: need 45 bevel to improve marginal adaptation

Resin etch technique: conserves tooth structure, reduces microleakage, improves esthetics, and provides micro-mechanical retention have best initial seal but decrease over time (amalgam gets better overtime)

Advantage of resin of amalgam: more conservative, but disadvantage: less wear resistance

Composite Filler Particle Functions: Decrease coefficient of thermal expansion & polymerization shrinkage. Increase tensile and compressive strengths, hardness, and improve wear resistance. (hybrid stronger than microfiller but micro smoother) (matrix UDMA, bis-GMA) and silane (coupling agent)

VCL: 470 nm (retinal damage)

Main cause of amalgam failure: delayed expantion b/c insufficient trituration & condensation, and amalgam contamination by moisture during trituration and condensation

Main cause of fracture of amalgam: not enough depth (number one failure in kids)

Gives the color to teeth: Dentin, Enamel (translucency)
protection for root caries? Fluoride gel in custom tray 0.4% Stannouss fl (causes staining) or neutral sodium fl 1% (most common over the counter) must be kept in for 4min (SnF acidic and bad for porcelain ph is 2)

Animal has caries and feed them cariogenic food via stomach tube. What will happen to the caries intraorally? Decrease, stopped, increase, unaltered

 

White spot lesions are:

Only Seen when dried with air-dried with air syringe

Subenamel porosities

Restored with Composite <- no, we don’t restore them.

Ans A

Which of these decreases the vertical development of dental caries?
a. Xylitol

b . chlorhexidine

Floss

Fluoride

Ans D

Green and orange discoloration of anterior teeth is due to?
A-Diet
B- Bad oral hygiene
C- Mixed Medication
D- Other options
Ans B

What is not internal line angle in a distal Class II: distoaxial

The means by which dental patients are treated to eliminate the caries process is called

A- Antibiotic prophylaxis

B- Primary prevention (fl, sealant)

C- Secondary prevention (rein, amalgam)

D- Tertiary prevention (rehab)

ANS: C

 

Smooth surface lesions resulting from flexure of the tooth structure are known as which of the following?

A- Abrasion

B- Erosion

C- Abfraction
D- Attrition
ANS: C

 

How many pins should you place in amalgam?

A.1 pin for each cusp

B.2 pins for each cusp

C.1 pin in proximal side

D.2 pin in proximal side

Ans A (1 mm from DEJ, 2mm from cavosurface, 2mm inside in dentine, 2mm )

 

Arrested caries:

A leathery brown

B Dark and soft

C Shiney back dark

Ans:C

 

What is the LEAST likely reason for postoperative sensitivity after a Class I occlusal composite restoration is placed?

A- Gap formation which allows bacterial penetration into the dentin tubules

B- Gap formation which allows an outward flow of fluid from through the dentin tubules

C- Direct toxic effects of a 15 second acid etc on the pulp

D- Cuspal deformation due to contraction forces of polymerization shrinkage

Ans D

 

Which of the following represents the most frequent cause of failure of dental amalgam restorations?

A- Moisture contamination

B- Improper cavity design

C- Improper condensation

D- Inadequate trituration

ANS: A failure is due to moisture contamination

 

Which of the following represents the most frequent cause of fracture of dental amalgam restorations?

A- Moisture contamination

B- Improper cavity design

C- Improper condensation

D- Inadequate trituration

Ans B: fracture is due to improper cavity design

 

What is true about c factor?

It is ratio of the unbounded to bonded

It is the lowest for class 1

With the increase in bonded surf, increase shrinkage

With the decrease in bonded surf, increase shrinkage

Ans. C it is the ratio of bonded to unbounded surfaces (highest in class I and class V)

 

Worst detection of caries?

A- X ray

B- color

C- caries detector

D- probe
ans D. probe

 

Which of the following would be LEAST likely to lead to the development of root surface caries on facial surfaces?

A- Low salivary flow

B- Elevated levels of sucrose consumption

C- Streptococcus sanguias dominating adjacent plaque

D- History of head/neck radiation therapy
ans C

 

All true of Strep. Mutans except?

Can live in plaque

Can live on gingival

Can live in a child with no teeth

Has to live on a non-shedding surface

Can live in a denture teeth

Ans. C

 

Each of the following is a cause of postoperative marginal ridge fracture associated with amalgam restoration EXCEPT one. Which one is this EXCEPTION?

A) Axiopulpal line angles not rounded

B) Marginal ridge left too high

C) Incorrect occlusal embrasure form

D) Improper removal of matrix

E) Under carving

Ans E (final carving done when band taken off)

 

you have mesial and distal caries on max 1st molar you decided to make it MOD rather than separated cavities because of

1 remaining width of oblique ridge less than 1.5 mm

2 for more retention

3 for resistance

4 to gain occlusal anatomy

Ans 1

 

22-Which of the following have the most effect on caries formation

1 ph salaiva

2 diet

3 specific bacteria

4 time

Ans 3

 

Pulp reactions to caries include all of the following, EXCEPT 

decrease in dentin permeability
B. tertiary dentin formation
C. evaporation of the intratubular fluid.
D. inflammatory and immune reactions
ans C

 

Most common tooth to have caries

1 max 1stpremolar

2 max 1 molar

3 man 1st molar

4 man 1st premolar

Ans 3

 

Least likely to have caries

1 man anteriors

2 max anteriors

3 man premolar

4 man premolars

Ans 1

 

Caries in radiotherapy patients MOSTLY in

1 cervical

2 occlusal

3 proximal

4 All

Ans 1

 

Which of the following is the single most important factor affecting pulpal response to tooth preparation?

A- Heat

B- Remaining dentin thickness

C- Desiccation

D- Invasion of bacteria
ans. B (from mastery)

Displacement of Odontoblastic processes is caused by:

a.Deccisation

b.Chemical

c.Mechanical

d.Thermal

ans A (causes fast out flow of fluid which causes pain)

 

32- Where you shouldn’t you put a temporary filling

1 have no time to finish treatment today

2 cusp of the molar is broken

3 emergency and need a root canal

4 pt has many caries lesions that has to be done today

Ans 2

 

The day after a routine Class V composite was placed, the patient reports discomfort from the tooth. Which of the following is most likely responsible for this complaint?

1)No liner or base used

2)Over-etching with phosphoric acid

3)Exposure of root dentin during the finishing procedures

4) too deep the axial depth of the preparation

ans 3

 

33 -Pt went to educational clinic with deep class 3 in upper lateral, while the student preparing the cavity the time is over so he should put temporary filling which one he should use

1 GIC

2 IRM

3 zinc oxide (cavite)

4 zinc polycarboxylate

Ans 2

40-year- old patient has 32 unrestored teeth. The only defects are deep- stained grooves in posterior teeth. The grooves are uncoalesced. What is the treatment of choice?

A- Periodic observation

B- Pit and fissure sealant (sealants arrest/stop incipient caries)

C- Preventive resin restoration

D- Application of topical fluoride

Ans a

 

 Patient has a new amalgam restoration, most likely experience in proximal hours:

Cold

Heat

Sweeteners

Galvanic shock

Ans A.

 

Why replace composite anterior
1 discoloration

2 marginal detach

3 fracture

Ans 1. (the most common reason to replace composite is caries, then discoloration. If caries, then this is answer but marginal detachment does not mean caries)

 

If a rubber dam is abnormally wrinkled between teeth, the probable reason is

A-overlapping or crowding of the teeth involved.

B-teeth with abnormally broad contacts.

C-the holes were punched too far apart.

D-the holes were punched too close together. E-the holes were not punched large enough.

Ans c

 

class 1 composite restoration

Pulpal depth 2 mm

Cavo-surface angle 45

Sharp line angles

Only do pit and fissures where caries is

Ans 4 (cavo-surface in composite should be obtuse greater than 90)

 

What is the correct method of excavation of deep caries close to pulp?

A-Large bur from periphery to the center

B-large bur from center to periphery

C-small bur from periphery to center

D-small bur from center to the periphery

Ans A

 

Indirect composite inlay has the following advantages over the direct composite EXCEPT:

Efficient polymerization.

Good contact proximally.

Gingival seal.

Good retention

Ans D (no dentinal undercuts**)

 

Which of the following shows least microleakage.

A ceramic inlay

B direct composite resin inlay

C indirect composite inlay

D direct composite.

Ans A

 

12-Polishing bur

1 more fluts less depth

2 Less flutes less depth

3 More flutes more depth 4 Less flutes less depth

Ans 1 (polishing: 12-30, 4-8: cutting, to polish composite with aluminum oxide burs)

 

Which material is good for both class II and class V cavity?

Amalgam

GIC

Composite

Zn phosphate
Ans A (in post class V GIC, in anterior composite)

patient present with 1.5 mm Diastema between #8 & 9, no carious lesion, what is the best

treatment for her.

Indirect composite veneer

Direct composite veneer

Porcelain veneers

Composite restoration in interproximal

Ans D ( with diasthema wait till canine out, recipricol anchorage with ortho and then cut frenum)

 

 

 

PROSTHODONTICs

We use type III semiadjustable articulator; can stimulate lateral, protrusive and bennet mvmt. Use facebow and intraoral Max-mand records

Acron Articulator: The condyle element is in the lower member, resemble most accurately the

TMJ anatomy, good for fixed prosthodontics

The average settings or values of the Condylar inclination on the articulator for sagittal and lateral condyle path inclinations horizontal condylar guidance 30 and (15 degrees bennet angle from non working side formed in saggital plane viewed in horizontal, working is bennet mvmt/side shift which is 30 HCG)

Curve of Spee: Anterior – Posterior curvature of the mandibular occlusal plane

Curve of Wilson: Mesio – lateral U-shaped curve of Upper & Lower posterior teeth

Contacts in Balanced Occlusion:

Cusp-to-fossa contact in centric occlusion in an ideal class I occlusion (reduce Mx L cusp tip in centric) ( Mandibular buccal is secondary centric holding cusps)

During lateral excursions: working interference BULL inner and LUBL outer

During lateral excursion: LUBL inner

protrusive interference: DUML (facial cusps)

centric interference (fwd slide): MUDL

MIC/CO: tooth guided position
CR: ligamentous guided position
VDR/interocclusal distance: muscles guided position

Posterior determinant of occlusion during protrusion: condylar guidance

Anterior determinant of occlusion during protrusion: incisal guidance

Canine guidance during lateral mvmts: by Canine on working and condyle on balance side

Anterior guidance: incisal +canine guidance (set with pin and guide table with acrylic resin)

Mutually protected occlusion: ant teeth protect post, post protect anterior

 

For denture pt we want group function or bilateral balanced occlusion (ANT GUIDANCE should be avoided in denture for bilateral balanced)

Protrusive record 3-6mm: measures condyle guidance and setting condylar angle on articulator

To set medial on superior condylar guides on arcon artifculator: take lateral interocclusal record

Christenson phenomenon: when posterior teeth open during protrusion, want to avoid in bilateral balanced occlusion (post opening increased in IG increased and with horizontal CG but IG effects front more and HCG posterior more)
hannau quint: CondylarG + Insical G = P of occlusion+ Comp Curve+ Cusp Heigth

CG brought by pt, out of our control but with articulator can max increase CG by 5 mm, so as CG increases so must CC

Compensating curve: under dentist’s control* Helps to provide a balanced occlusion

In centric position: bilateral balanced occlusion. All cusps are integrated

More prominent CC required when theres steep CG assoc with low IG

Occlusal Plane: tip of Ant teeth, post should be 2/3 of retromolar pad height
cusp angulation: usually 30 or cuspless

condyles go in CR: Antero-superior with thinnest avascular portion of disc

CR records: SHOULD NOT BE PERFORATED, confined to cusp tips (worst is soft wax)

IOR-interocclusal reccord: show have min resistance to jaw closure and low flow mixing (for aginate casted use wax, for elastomeric casted use PVS)

camper line: tragus to ala of nose (use fox plane with this) sets max occlusal rim (parallel to interpupillary)

Flabby tissues in anterior maxilla for a complete denture impression = Passive/mucostatic
lingual flange of mandibular denture is determined by = Mylohyoid

question about working and non working interference? understand the inclines for each side.

Lingual flange of mandibular denture (molar area) is determined by: Mylohyoid

Mandibular flange (lateral to retromolar areas) is determined by: Masseter

 

 

COMPLETE DENTURE:

Primary support for mandible: buccal shelf, limited laterally by external oblique ridge secondary support for mandible: alveolar ridge (retromolar pad/basal bone adds support and retention and doesn’t resorb)
Primary support for maxilla: residual ridge and palate secondary support for maxilla is ruggae

Stability: ridge height and vestibule

Stability of denture: flange, occlusion, musculature

Support of denture: base

Retention maxillary: palatal suction from vaccume provides peripheral seal

Retention mandible: covering extensive area & vestibule extention and lingual seal

past post palatal seal: dislodge and drop. If terminate anterior to this, on the hard palate, no seal is created, the denture is unretentive. Median palatal suture/torus needs relief

Custom tray 2mm short for boarder molding (use ZOE all in one go or modeling compound)

Impression: most important is stability and lack of displacement

Boarder molding for maxilla: **behind max tuberosity DB by hamular/coronoid notch,

Boarder molding for mandible: *DB corner by masseter, DL superior pharyngeal constrictor, retromylohioid area: palatoglossus and SPC, mylohyoid, anterior lingual genioglossus and mylohyoid, sublingual gland labial by mentalis(extention) and labial frenum(thickness), buccal vestibular by orbicularis and buccinators and depressor anguli
posterior palatal seal: Anterior boundry (Valsalva butterfly) posterior boundary (vibrating line usually 2mm in front of fovea delineates hard and soft palate) between these is post dam

post dam: anterior boundry V groove, scribed 1.5 in base and 1.5 in height so account for shrinkage porosity, palatal glands help with peripheral seal and retention, it is in immovable tissue. If excessive post dam thickness, denture unseats

flatter the palate, the flatter post dam

Bead and boxing of adequate width for cast = 0.75mm or 3/8” Cast thickness = 16 mm from the highest spot on the and 5 mm from sides

Tingeling at lower lip: reduce buccal flange (mental foramen impinged)
tingling palate: reduce incisive papilla

Soar throat: impinging retromyloheioid (SPC and palatoglossus)

Ant max teeth should be set are 8-10 mm infront of max papilla so facial to ridge

Cheek biting: cause by horizontal edge to edge of teeth

Clicking: increased VDO (check s sound)

Trouble swallowing: inc VDO, dec interocclusal space

Biting corner of mouth: canine and premolar set too fwd

Biting lip: increase horizontal overlap

Tongue biting: post teeth set too lingually

Ridge soar: premature occlusal contacts (use disclosing wax to see)

Dislodge mandible: if teeth on ascending ramus, if OVEREXTENDED DB flange (masseter)

Dislodge maxilla: if corornoid area too thick, if post dam too deep.

Gagging: bad palatal seal, too far back, increased VDO

whisteling S sound: palatal arch too high or narrow, vertical overlap not enough, too much horizontal overlap
LISTHP: incisor too far back, palate too thick
max and mn premolars contach during sibilant sounds tx: fix VDO

take face bow to locate hinge axis point only (needed if we wanna increase VDO preserve on articulator with plaster index (preferred) or 10x wax. Can increase by 5degree)

Children with dentures, pagets and acromegaly may need dentures changed often.

Pts w debilitating disease need: maximum extention, narrow occlusal table, passive impression, no porcelain, good occlusion, reinforce OH, recall 6 mnth min

Reline for immediate : 5 and 10 month
evaluate after entrega after 24 hr (hypersalivation is normal)
occlusal disharmony: must remout in CR and adjust out of mouth
porosity in denture: insuff pressure on flask, rapid elevation in temp, packing and processing of sandy acrylic

patient has problem in swallowing, while making an impression for complete denture … which area in mandiblular denture is hard to register? Masseteric notch

 

Best to preserve root supported over denture

1 daily fluoride

2 metal coping

3 amalgam post

Ans 2 with stannous fl 0.4% gel

 

would relieve a mandibular denture in the area of the buccal frenum to allow which muscle to function properly?

Buccinator

Orbicularis oris

Triangular (and buccinator and orbibularis)
(mandibular labial flange is limited by mentalis muscle)

labial frenum – orbicularis oris for lower (for upper nothing)

for upper buccal it is levator anguli, buccinators pulls back, orbicularis pulls fwd

 

Each of the following governs the extension of the buccal flange while making a mandibular impression on an edentulous arch EXCEPT one. Which is the exception?

1) Buccinator muscle

2) masseter muscle

3) buccal shelf

4) External oblique ridge

5) Mucobuccal Fold
Ans: 4

 

Primary support for complete denture?
A- Rugae in maxilla & Buccal shelf in mandible
B- Alveolar ridge of both maxilla & mandible
C- Other options

Ans C

 

Management of epulis fissuratum/inflammatory papillary hyperplasia is very important because:

Its very painful

It has neoplastic activity

It affects denture retention

It affects denture stability

Superinfection is a regular complication.

Ans D (stability=flange/stable base)
TX of hyperplasic tissues may involve: tissue rest, soft relign, changing habit (taking out at night), but ultimately need surgical removal if extensive

 

Management denture stomatitis (candida) is very important:

Its very painful

It has neoplastic activity

It affects denture retention

It affects denture stability

Superinfection is a regular complication.

Ans A

 

 

Each of the following is a common cause of denture gagging EXCEPT one. Which is the EXCEPTION?

A- Inadequate posterior palatal seal

B- Excessive vertical dimension

C- Bulkiness of denture

D- Excessive anterior guidance

Ans D.

 

In developing balanced occlusion of complete dentures during tooth arrangement, a steep condylar path associated with a low degree of incisal guidance requires that the compensating curve be

flat.

prominent.

shallow.

4.reverse.

 

 

Lower Denture flange distal extension should

1 Cover 1/2 or 2/3 of retromolar pad

2 cover all retromolar pad

3 do not cover

Ans 2 (but if talking about occlusal rim only 1)

 

On delivering cast restoration the sequence shod be used: check internal

fit then proximal contact then internal form, marginal integrity and last occlusion

 

An edentulous patient, and you want to measure VDO what sounds?
S (occlusal rims should barely touch)
checking plane of occlusion with?

Clicking of the dentures during speech most often indicates which of the following?

A- Inadequate denture retention

B- Decreases occlusal vertical dimension

C- Insufficient interocclusal space

D- Improper buccolingual position of the teeth

ANS:C

In a maxillary complete denture opposing a mandibular bilateral distal extension, Why is the anterior of the wax rim of maxillary beveled?

A- VDO and length of maxillary occlusal rim was adequate .

B- VDO was incorrect but length of occlusal rim was adequate

C- always bevel Max occlusal rim

D- length of occlusal rim is adequate for esthetics but VDO was wrong

ANs b

 

Patient 74 years old wants new total dental prosthesis upper and lower (DAY 2)

-Anatomic with 20 grade of cusps angulation

-Anatomic with 30 grade of  cusps angulation

-Semi-Anatomic with 10 grade of cusps angulation

-Non Anatomic with zero angulation

Flabby tissues in anterior maxilla for a complete denture impression = Passive/ mucostatic technique

In selection of maxillary teeth for overdenture abutments, the ideal location is:

A.Maxillary canine

Maxillary lateral incisors

Maxillary premolars

Maxillary central incisors

and D (to get tripod effect. Its CI and Canine for upper, C and PM for lower)

 

First sign of increase occlusion (VDO)

TMJ

Myofacial

Attrition

Abfraction
Ans: B (will have myofacial pain from strain of closing muscles, trauma to underlyeing tissues, decreased freeway space)

combination syndrome shows all except

A.maxillary ridge resorption

B.flabby tissues

C.enlarged tuberosities

increased VDO

Ans D
Combination Syndrome: decreased VDO, flabby hypoplasic max ant, hyperplasic tuberosity

 

Minimum interocclusal space b/w max ridge/mandibular ridge: 3mm, if less, need qx
Which of the following is the most important factor in determining patient satisfaction with dentures?

A- Dentist-patient relationship

B- Bone height for denture fit

C- Patient personality traits

D- Technical quality of the denture

E- Cultural definitions of esthetics

Ans C

in non-working lateral interference which muscle can possibly experience spasm?
1-Medial pterygoid
2-lateral pterygoid
3-Masseter
4-Temporalis

You will be constructing a new maxillary complete denture and a new mandibular overdenture for an edentulous patient. Why is the denture construction recommended prior to surgical implant placement?
A. The denture can be used as a guide for location of the implants.
B. Improved fit.
C. Improved occlusion.
D. Establishment of vertical dimension of occlusion.
E. All of the above.

Ans all

Which of the following refers to a decreased occlusal vertical dimension?

A- Vertical dimension that leaves the teeth in a clenched, closed relation in normal position

B- Occluding vertical dimension that results in a excessive interocclusal clearance when the mandible is in rest position

C- Insufficient amount of interarch distance because of the bony ridges

D- Condition in which the patient cannot open mandible because of temporomandibular joint pathology

Ans B (VDR-VDO=FS 2-3mm)

 

PROSTHODONTICS: FPD

Ante’s Law: Root surface of abutment teeth have to be greater than root surface of Pontic, the longer the FPD, the poorer the prognosis

Disadvantage of cantilever bridge? Rotational forces

deflection of Pontic occlusogingival: directly related to cubic Pontic length

minimal accepted C:R ratio: 1:1
packing cord: Epi (avoid w/ HTA causes inc in BP), aluminium potassium sulfate, Zinc Chloride (causes necrosis of sulcular epi), electrosurgery (removes thin layer of sulcular epithelium contraindicated in pt w pacemaker/insulin pump)

Gold Type I & II: Used for Inlays
Gold type III: Used for all metal ceramic crowns & Bridges

Taper: 2-5 per axial wall

 

The best pontic design for best papilla support and superior esthetics is:

A- Ovate

B- conical

C- adjusted saddle

Ans A (but reqiores surgical prep, 2nd best is modified ridge lap)

The strength of a soldered connector is best increased by
A- Using a higher carat solder
B- Electroplating the joint with gold prevent corrosion
C- Increasing its dimension in a direction parallel to the applied force
D- Increasing its dimension in a direction perpendicular to the line applied force
E- Increasing the with of the joint by having a space of at least 0.5 inch between the parts to be soldered
ans c

Post: parallel sided more retentive than tapered, threaded more retentive than smooth

Patient with lateral incisor, RCT, metal core, post and PFM. ferrule this tooth is for:

1- Retain the core

2- To hold the tooth and prevent fracture

3- prevent rotation of post and core
Ans 2 (The ferrule (or height) of the preparation providing friction and retention (1.5 mm vertical sound tooth structure minimal above finish line circumferentially). Ferrule effect is envelopment of tooth structure by crown to prevent root fracture. if less needs core-buildup, Cr Lengthening or both)
5 mm of suprabony tooth structure (is ferrule 1.5+core 1.5=length of wall minimum 3mm + biological width 2) =4-5

 

A FPD with single Pontic deflected certain amount, a span of two similar pontic will deflect:

Same amount

Twice as much

Four times as much

8 times as much

Ans D (its Pontic number cubed so if 2 Pontic answer is 8, if 3 pontics answer is 27)

 

Primary reason for doing a 3/4 crown?

EPT

Accessible for cleaning

Tooth structure preserved

Easily seated during cementation

Ans 3

 

What cement to use for cementation of a Laminate porcelain veneer?
a. Adhesive resin

Other option

Glass ionomer

RMGI
Ans A

Best root support FPD

1 long less bone loss

2 Short less bone loss

3 Conical more bone loss

4 Conical less bone loss

Ans 1

Lab over bulks porcelain, why?
Not enough reduction on tooth (most common complain of lab is not enough reduction)

Auxilliary resistance from features in fixed dental prostheses such as boxes and grooves should ideally be located?

A- Facially

B- Lingually

C- Occlusally

D- Proximally

Ans D ( for retention buccal A for mandible, palatal for max, wherever theres most bulk)

 

which abutment/cantilever system will have the most traumatic effect on the abutment?

molar abutment and premolar Pontic

premolar abutment and molar Pontic

C) max lateral abutment, max central pontic

D) “max central abutment, max lateral Pontic
ans c. (conical shape of root will make it torque more but distal cantilever molar also bad)

 

123- What not to do in veneer:

1.try in paste for shade

2.apply silane to inner surface

3.apply bonding agent

4.etch enamel with hydrofluoric acid

Ans 4

 

Provisional for veneer? Bis-acryl
What cement to use for cementation of a Laminate porcelain veneer?  Light cure Adhesive resin

101 – how much the reduction from incisal edge in veneer

1 0.3mm

2 0.5mm

3 1.0mm

4 1.5 mm

Ans 3 (0.3 gingival, 0.5 midfacial)

btw need to leave 4mm of GP for apical seal or 2/3 of root length

posts should always be below 1/3 of tooth M-D width

Post: parallel sided more retentive than tapered, threaded more retentive than smooth

 

pin-retained restauration, the pin should be parallel to:
A-long axis of the tooth
B-nearest external surface
C-pulpchamber
D axial wall (2mm of dentine between DEJ and post, 2mm into dentine and 2mm into restoration)

most retentive post? 1-tapered 2-parrallel 3-serrated 4-smooth

 

130- The function/effect of post in post and core
A. Provide retention for a crown
B. Enhance the strength of the tooth

Provide retention for a core

Provide the root canal sealing
E. ferrule effect

Ans C

What angulation post would be placed for retention

Taper

Long axis

Vertical

Follow the canal
ans: prefabricated metal posts: active (threaded) vs passive. threaded bad. parallel and tapered preferred. parallel has better retention, but requires more dentine removal, than taper. (height should be min 6mm) fiber-reinforced are not cemented, rather bonded (better retention)

what is the factor that prevent the fracture of tooth with post n core:
A. remaining tooth structure
B. Post length
C. Post width
ans. A

advantage of using a fiber-reinforced (quarts, carbon fiber) post for restoring an endodontically treated tooth:

A- Has a modulus of elasticity similar to stainless steel

B- Has a modulus of elasticity similar to dentin

C- Is highly radiopaque and easy to visualize on a radiograph

D- Is stronger and more resistant to fracture than a cast metal post
ans B

 

Glass-fiber vs. custom made post:

A – Better adaptation

B – Increased Esthetic

C – Require less tooth structure removal

D – Provides better resistance to tooth structure

Ans: all of the above
safer, more easily removed, aesthetic, conserve tooth structure, and provide improved fracture resistance to these compromised teeth but disadvantage is that they may not be able to withstand flexural resistance against core

 

 

RPD:

Support: resist vertical (occlusion) into tissue or abutment, stability resists (horizontal/torque lack of displacement from accurate seal/seating), retention resist removal away(sticky)

Support: abutment occlusal rest and residual ridge
Stability: occlusal harmony,reciprocal clasp, proximal plate/guide plane (guide path of insertion should be 1/3 of BL width and 2/3 of GO hieght)

reciprocation: guidance planes (during insertion and removal), reciprocal clasp, minor connector

major connector and minor: cross arch stability and rigidity. distribute stress.

denture base: support

retention: retentive clasp

retainers: provide support and retention
indirect retainer (Canine or PM): rest seat + minor connector: located as far anterior as possible, opposite of fulcrum line, fcn prevent vertical dislodgement of the distal extension base of RPD, need in kenedy I and II, don’t need in kenedy III, protect soft tissue from major connector . prevents rotation around the fulcrum line

direct retainer: rest seat + minor connector+ clasps (recipricol and retentive)

RPI: mesial rest, I bar, proximal plate

Occlusal Rests: min 1-1.5 at marginal ridge and min 0.5-1 mm deeper at apex of triangular fossa, spoon shaped, concave. Must be 1/3 of F/L and 1/2 of width from cusp tip to cusp

Cingulum rest: inverted U M-D width 2.5, FL depth 2, GO height 1.5 (less torque over butment and more esthetic than incisal)
rest with minor connector angle: less than 90 (beading of maxillary cast, for major connector adds rigidity and allows more contact with palatal tissues to stop food)
clasp assembly: retentive clasp + reciprocal/stabilizing clasp+ minor connector+ rest

Reciprocal claps: stabilizes (above or at HOC) lingual, touches tooth before or at same tie as retentive clasp. Covers 180 degrees.
retention clasp: retention(below HOC terminal 1/3 is wrought wire more flexible contacts gingival 1/3) buccal. At seat should be passive, applying no pressure. Wrougt wire is 0.02 inches (cast clasp is 0.01inches)

Survey line: to help find path of insertion, undercuts, and HOC (type I. deepest undercut is in portion of tooth AWAY from edentulous space. Type II. Deepest undercut TWD edentoulous space. Type III. Deepest undercut may be anyways below survery line)

Realign when pressing on base and indirect retainer lifts.

beading along border of max major: 0.75-1mm deep

Corrosion of Nobel metal: chromium (high nobel metal is corrosion/tarnish resistant)
most people allergic to: nickel in RPD

 

Minimum clearance for occlusal rest is 1.5 mm clearance (this is what you check with wax)

occlusal rest MINIMUM 2 in center and 1.5 mm minimum on marginal ridge. Must be 1/3 of F/L and ½ of width from cusp tip to cusp concave, spoon shaped, rounded.

 

Patient with partial interim denture, how is it different:

esthetics

retention

resistance to occlusal loading

Ans C (no rests so vertical support is effected)gonna have retention from clasps and esthetics

If patient has gagger and inoperable palatal torus and lingual has less than 7mm of space to floor:

A-P

Horshoe(least rigid)

Palatal strap

Full palate plate

Lingual bar

Labial bar

Lingual plate

 

When you push on distal extention of lower RPD, and indirect retainer rest comes up, tx?
a. Relign

tell them to use adhesive

adjust clasps
ans A (press base and IR to relign)

 

I-bar, T clasp in RPD, in which direction will it move during function?

a) Gingival and mesial

b) Gingival and distal

c) Occlusal and mesial

d) Occlusal and distal

 

The principal function of an indirect retainer in distal extention RPD is to

stabilize against lateral movement.

prevent settling of the major connector.

minimize movement of the base away from supporting tissue and minimize movement of the base towards from supporting tissue.
and 3

 

Major connector: rigidity and stability (beaded to depth and width grove of 0.5mm, increases strength and prevents food impaction)

Mandibular bilateral distal extension for long time, when you put pressure on one side, opposite lifts:
a. no indirect retention used

rests do not fit

acrylic resin base support

Occlusion

ans (rocking RPD = no IR)

 

Patient complains that his new bilateral distal extension RPD “feels loose” and abutment tooth is sensitive to percussion issue?

A- inadequate indirect retainers

B- inadequate seating of denture

C- the retainers are passive on the abutments.

occlusion
Ans D

 

Patient complains that his new bilateral distal extension RPD is rocking. What is problem?

A- inadequate indirect retainers

B- inadequate seating of denture

C- the retainers are passive on the abutments.

occlusion
Ans A

 

Purpose of major connector

A- stability and rigidity

B- stability and retention

C- retention and rigidity

D- rigidity and esthetic

A (major connector provides rigidity and cross arch stabilization)

What is combination clasp: type of circumferencial clasp where retentive arm made of wraught wire for flexibility, reciprocal arm is ridged from metal cast, and distal rest

 

126- – Most common cause of rest breaking?

A.Heavy occlusion

wear by opposing tooth

C.inadequate rest seat prep

Ans C

 

121- When placing I bar on premolar distal extension, under occlusal forces the i bar moves

A-Occlusally and distally

B-Occlusally and mesially

C-Apical and distal

D-Apical and mesial

Ans D

 

 

DISEASE INVOLVED WITH BACTERIA/VIRUS:

Primary bacteria for initiation of caries: Strep. Mutans

Red Complex Bacteria: P. gingivalis, T. forsythia, T. denticola

Oral hairy leukoplakia: EBV (not premalignant)

Papilloma, Condyloma Accuminata(reoccurs): HPV 2, 6, 11, 16,18 (highly recurring)

HPV assoc with cancer: HPV 16, 18

Hecks:HPV 13, 32

Koplik’s Spot: measles, rubeola

Ramsay Hunt Syndrome: caused by Herpes Zoster. Associated with shingles, facial nerve

damage, & loss of hearing in affected site

Patient with a White coating of the tongue that sloughs off leaving a deep red surface with swollen hyperplastic fungiform papillae. Diagnosis: Scarlet fever. Key word fungiform

 

 Scarlet    Fever    symptoms    except:
A.Fever
B. malaise
C. skin rash
D. lymph swelling
E. red inflamed tongue
F. strawberry gingiva
Ans F (F seen in wegener’s granulomatosis)

EMERGENCY MANAGEMENT:

Most common respiratory emergency in dental chair: Hyperventilation related to anxiety/panic

Symptoms: Tachycardia, Tachypnea, dizzy, chest pain, fainting, carpopedal spasm, metabolic alkalosis, hypocapnia

Tx: sit up, DO NOT GIVE O2

Most common resp after IV anesthesia is: atelectasia (24 hr after), asp pneumo, pulm embolus

If asfixating: cyanosis+stridor hiemlich or if not bad x-ray/hospital

Most common dental emergency (90%): Syncope (vagovagal or psychogenic)

Symtoms: pale, diaphoresis, nausea, bradycardia, and HYPOTENTION, dialated pupils

Tx: check if they are awake, trendelenberg, head tilt/chin lift, O2 administration, place cold compress on head. If unconscious CAB* (circulation, airway, breath, CPR always 30:2 so 100 compressions, 12/20 breaths)

Acute asthma symptoms: cough**, chest tightness, dyspnea, tachypnea, episodic wheezing

Most effective during acute asthmatic attack: sit up, terbutaline/ albuterol (beta-2 agonist) and O2

Medication for status asmaticus: Aminophylline (bronchodilator), Albuterol (bronchodilator), corticosteroids (for Long term asthma)

Contraindicated drugs in asthma: avoid Aspirin, NSAIDs and narcotics, use mepi w/o vasoC those on beta-agonists and in theophylline avoid macrolids.

Anaphylactic shock symptoms: hives, rash, prurutis, angioedema, stridor
For acute rxn tx: diphenhydramine IV
if you hear stridor= laryngeal obstruction always administer O2 first
Sever: dose of epinephrine in anaphylactic shock: 1:1000 (0.3 mg IM), call 911
Patient Positions:

Syncope: Trendelburge

Left lateral decubitus for prego: relieve IVC from baby (15 degree hip up)

Upright: Asthma, COPD, postural, crown in mouth

Supine: seizure

Most common seizure in kids: Febrile

tx of seizure:

grand mal: phenytoin/ Dilantin (most common type after febrile)

petit mal: ethoxamide or valproic acid

status epilepticus: IV diazepam

Most common heart condition in child: Ventricular septal defects

Hypoglycemia symptoms in diabetic: pallor, diaphoresis, tachycardia, hunger, confusion, agitation, coma, hunger, lack of coordination
Unconscious diabetic is treated with: 50% dextrose in IV, 1mg glucagon IM
diabetic and general anesthesia: clear liquids 2 hrs before, light meal 6 hours, heavy meal 8 hours (American society anesthesiologists fasting guidelines) ½ insulin
Angina: chest pain referred pain to left arm, 10 min
Tx of angina: (ONA): stop, position upright, O2, NTG 0.4mg spray/tablet, reassure, take vitals, NTG if cont pain after 5 min, NTG 3rd dose + chew asprin+911
Anti-anginal Drugs: NTG, verapamil (ca ch blocker), proponolol
MI: SOB, cool skin, tachycardia, diaphoresis, hypotention
Tx (MONA): morphine, O2, NTG, asprin
angina and MI how to differentiate: duration longer in MI, Pain could be more intense, other symptoms more common in MI
MI: thrombosis and arrythmia: defibrillator
LA toxicity: intravascular injection and too much: numbness, biphasic early CVS/CNS stimulation (tachy, HTA, agitation, slurred speech, tinnitus, metallic taste) later CVS/CNS depression (hypoT, bradyC, unconsciousness, seizure, ventricular dysarythmia, coma)
Tx: diazepine IV
LA allergy: esthers (PABA), methylparaben (preservative)

Patient with chest tightness and ache going to left arm, first step?

A- Oxygen

B- Aspirin

C- Raise chair from supine to upright

Ans. C always in angina: stop tx, position, O2, NTG, reassure, take vitals

 

How to treat patient with PTSD?

midazolam every time

explain to him/her everything (sort of tell-show-do)

Ans A

 

Dentist applied topical benzocaine on patient mouth. Patient got pale discoloration, cyanosis, chocolate brown blood, what is the reason

Allergic reaction

Hypertension

Methaglobenemia
ans. 3 causes of methamoglobulenemia: LA: prilocaine/lidocaine, benzocaine, antihypertensive: Amyl Nitrite, acetaminophen at high doses

 

You put Pregnant patient in left position To prevent the pressure on

1 fetus

2 IVC

3 SVC

4 bladder

Ans 2

 

77- Patient has prosthetic heart valve, penicillin allergy which you give:

600mg clindamycin 1 hour before

2 mg amoxicillin 1 hour before

600 mg clindamycin 1 hour prior

No need

Ans 1 (2g(50mg/kg amoxi, 600 (20mg/kg) clinda)

 

Patient has facial swelling with 101 fever which antibiotic could you give

1 g ampicillin then 500 mg * 4 for 7 days

2 g amoxicillin one shoot

2g metronidazole then500mg *4 for 7 days

no need

ans 1(actyinomycosis is 14 day tx, 10000units )

 

Patient Blood pressure was 178/109. What is the most likely emergency this patient would have in the dental office?

Syncope

Hypertensive crisis.

Hyperglycemia

Hyperventilation.
ans b (leading to angina/MI/stroke b/c of aterosclerosis)

 

patient has uncontrolled HTN. What is the possible emergency situation might arise during dental treatment

A- stroke (transient ischemic condition)

B- syncope

C- hyperventilation

D- shock

Ans A (for stroke tx is: IV tPA-alteplase)

 

Patient needs premeditation for what conditions

A- heart murmur w/ regurgitation (medium risk)

B- replacement of aortic valve (high risk: prosthetic/ surgically constructed heart valve, previous IE)

C- cardiac pacemaker (negligible)

D- mitral valves prolapse (only with regurg is medium risk w/o is minima; risk)

Ans B

 

patient came for treatment. Patient is anxious prior treatment start. Pt complained about getting sensitivity or pain. During treatment, patient felt tingling in fingers and get unconscious. What happen to the

MI

Hyperglycemia

Hyperventilation

Thyroid storm (fever, agitation, adrenergic effects)

pheochromo (alpha adrenergic affects like headache, tach, diaph, but episodic and brief)

Ans C hyperventilation causes carpopedal spasm (tingling or pain of finger)

 

Slurred speech where?

1 Transient ischemic attack

2 dementia

3 2 more options

Answer is 1

ORAL PATHO

Systemic condition associated with endocarditis & glomerular nephritis: SLE

Gardener’s syndrome features: unerupted teeth, retained deciduous teeth, impacted

permanent teeth, multiple osteoma + intestinal polyps

Mechanism of Fetal Alcohol Syndrome: Neural crest apoptosis, mid face discrepancy

Most common age Primary Herpetic Gingivostomatitis: Age 1-5

Most common inherited: 1:700 Downs

Most common abnormality among the following: Cleft lip, then Cleft palate then Dentinogenesis imperfecta, Amelogenesis imperfecta, Dentinal dysplasia
Most common Cleft lip: in male, cleft palate: in female

Most commonly associated with dysplastic cells/dysplasia: Erythroplakia

Most commonly associated with osteogenesis imperfecta: Dentinogenesis imperfecta I

common between Crohn’s, Peutz-Jeghers & Gardner’s syndrome: Intestinal polyps

Gardner’s syndrome AD, rare= familial colorectal polyposis (adenocarcinoma of colon)+ multiple large epidermoid cysts+ 3-6 mandibular osteomas (radioO seen at puberty in angle) +multiple desmoid tumors (fibromatosis)+ prevalence for odontomas, supernumerary teeth, and impacted teeth + retinal abnormalitys + earl onset puberty
Peutz-Jeghers syndrome: multiple melanotic macules or pigmentation, GI polyps

Ectodermal dysplasia (x-linked recessive): sparse hair, anodontia (partial/complete), oligodontia, hypodontia, anhidrotic, conical shaped teeth
Common features of Cleidocranial dysplasia: Delayed eruption, supernumerary teeth

Peutz Jeghers syndrome: Pigmentation of face, lips, and oral cavity and intestinal polyps

Treacher Choline syndrome (Mandibulofacial dysostosis): relation to Zygoma

Cleidocranial dysplasia: relation to clavicle

Brown tumor (Central giant cell granuloma) is associated with: Hyperparathyroidism

Most common location of oral cancer in USA: Tongue

Pemphigus vulgaris: Suprabasilar vesicles and acantholysis

Pemphigoid: Subepidermal and NO acantholysis (hemidesmosomes, BM)

Papillon Lefevre Syndrome: Hyperkeratosis of palms & soles of feet, premature tooth loss,

Periodontitis

Patient refers to pain during swallowing and moving the head to the affected side dx:  Eagle syndrome (take a pano)

Crouzon syndrome: Beaten metal appearance of the skull. Hypertelorism (Increased interpupil

distance), mid face deficiency, cranial bones fuse too soon

Steven Johnson syndrome: Disease of skin & mucous membrane, begins with flue like

symptoms, top skin layer dies & sheds off, burning eyes

McCune Albright syndrome (Polycystic fibrous dysplasia): Café Au lait spots, Coast of Maine +polyostotic fibrous dysplasia +hyperPTH

­risk of osteosarcoma: McCune Albright, pagets
Associated with oral melanin pigmentation: PJ, addisons’s disease, McCune Albright, NFM I, smoking/racial/physiological pigmentation, cloroquinone, estrogen, or metastatic malignant melanoma

Neurofibromatosis 1: neurofibromas on tongue+Café au lait, Liche nodule of Iris, crowes sign

Plumer Vinson syndrome: atrophy of gastric and pharyngeal mucosa, spoon nails (Koilonycias)

(predisposal to oral SCC in postmenopausal females)+ iron deficiency

Frey’s syndrome: Gustatory sweating while eating and crocodile tears (parotidectomy)

Melkersson Rosenthal syndrome: Facial paralysis, cheilitis granulomatosis, scrotal tongue

SLE Lupus erythematosus affects in: heart, renal

Bechet’s disease: herpetiform Aphthous ulcer

Scleroderma: Mona-Lisa face, Widening PDL, limited open, purse sting mouth, deposition of collagen in organs lead organ failure, loss of mandibular ramus.

CREST syndrome: Limited Scleroderma, only in lower arms & Legs, sometimes face & throat.
 Most common location of SCC (most common oral cancer): Posterior lateral border of tongue (intraoral site)

Uncontrolled diabetes inhibits osteoblastic activity

Most common site of Basal cell carcinoma: Middle third of the face

Multiple myeloma: Bence jones protein, punched out lesion, plasma cell infiltrate, skeletal

radiolucency (Bone pain is the 1st sign)

Fibrous dysplasia: Ground glass appearance

Verrucous carcinoma (on vestible): Cauliflower, warts caused by HPV 16 & 18 and betel quid
Proliferative Verukiform Leukoplasia (tongue): poor prognosis, no tx

 

All of the following are congenital except…

dentinal dysplasia

amelogenesis imperfecta

regional odontodysplasia
d. ectodermal dysplasia
ans C.
Regional odontodysplasia/ghost teeth: enamel, dentin and cementum all affected. may be local vascular problem causing teeth to be affected during formation

x-ray: almost invisible teeth. Exhibit enlarged pulps, short roots, open apical foramina, thin enamel and dentin. Permanent maxillary ant. Tx: ext

 

CYSTS AND TUMORS:

Most recurrence cyst: KCOT/OKC (associated with Gorlin goltz syndrome)
Least recurrence tumor: AOT Adenoid odontogenic tumor or Compound Odontoma s.s

OKC is most commonly associated with: Nevoid basal cell Carcinoma

Nevoid basal cell carcinoma (Gorlin Syndrome): seen multiple OKCs and palmar pitting, planter kerato-cyst, causes cyst in the Jaws

Most common cyst in oral cavity: Peri-apical (radicular) cyst

odontoma: more common dontogenic tumor

Ameloblastoma: 2nd most common Odontogenic tumor

odontogenic myxoma/myxofibroma: most common odontogenic tumor of mesenchymal origin (3rd over all)
DD of mandibular posterior: ameloblastoma, KCOT, CGCG, CEOT

Most common non-odontogenic cyst: Nasopalatine duct cyst (X-ray: Heart shaped near central

incisor, tx: Enucleation)

 

What condition has multiple osteomas, GI polyps, and Skin cysts
A. Peutz-jeghers syndrome
B. Gardner syndrome
C. Crohn’s disease
D. Cleidocranial dysplasia
ans B. Gardner’s syndrome AD, rare= familial colorectal polyposis (adenocarcinoma of colon)+ multiple large epidermoid cysts+ 3-6 mandibular osteomas (radioO seen at puberty in angle) +multiple desmoid tumors (fibromatosis)+ ­prevalence for odontomas, supernumerary teeth, and impacted teeth + retinal abnormalitys + earl onset puberty

Least recurrence tumor

OKC

Compound odontoma

Ameloblastoma

Ans B (most reccuring is OKC)

 

Which cyst is not in bone?
A- nasolabial

B- Peripheral giant lesion

C- Ameloblastoma

D- Other options
Ans A

Which would NOT be included in a differential diagnosis of the right mandibular molar radiolucency?

A- Keratocystic odontogenic tumor KCOT

B- Ameloblastoma

C- Periapical (lateral radicular) cyst

D- Lateral periodontal cyst
ans: D (only D is between Mn C and PM)

 

A newborn girl was delivered via cesarean section due to airway patency concerns. During ultrasound, there was the discovery of a tumor of the oral cavity. On delivery, the pink, compressible tumor of the anterior maxilla was deemed to be a congenital epulis of the newborn. This lesion is composed of cells that are identical to those of the:

(1)a traumatic neuroma

(2)a schwannoma

(3)a granular cell myoblastoma

(4)a lipoma

3

 

23-Which of the following has no radiOpacities

1 AOT

2 fibrous dysplasia

3 ameloblastoma

4 condensing osteitis

Ans 3

Case of young patient, like 14 years old that has swelling on Maxillary canine area, not painful, few months of development, radiolucent image with fleck radiopaque. Diagnose?
a. Adematoid odontogenic tumor AOT
b. ameloblastoma
c. Calcifying epithelial odontogenic tumor/pindborg(CEOT in older people and post ramus)
d. other
ans a. Adematoid odontogenic tumor (AOT): 3-7% of all odontogenic tumors. Benign.

Clx: child/ teen, female, slow-growing, asymp unerupted/impacted MX C
x-ray: well-defined unilocular radioL surrounding crown of MX C apically than CEJ (DD: dentigerous cyst) with  “snowflake”* calcifications tx: capsulated so enucleation

 

Ameloblastoma is a most aggressive & most common EPITHELIAL odontogenic tumor. solid, well-defined, multicystic or polycystic (“soap bubble”) lesion, most aggressive kind.
TX: requires surgical excision

Ameloblastic Fibroma: compared to ameloblastoma – younger age, slower growth, does not infiltrate. Usually associated w/ impacted teeth

What is the most definite way to distinguish ameloblastoma from KCOT/KOC?

Smear cytology

Reactive light microscopy

Reflective microscopy

(multiple OKC seen in gorlin golz, benign could turn malignant, tends to reccur)

 

Radiographs reveal a radiolucency where the right third molar usually resides. The mandibular right third molar is not present. Which of the following should NOT be included in a differential diagnosis?

1)Odontogenic keratocyst

2)Odontogenic myxoma

3)Dentigerous cyst

4) residual cyst

Ans 3

 

If mucous glands are seen in the epithelial lining of a dentigerous cyst, this is called:

anaplasia.

metaplasia.

dysplasia.

neoplasia.

hyperplasia.

Ans B


Which one indicates a Stage III of the oral and oropharyngeal cancer?

A.Tumor less than 4 cm

B.Metastases in a single ipsilateral node 3 cm or less

C.Metastases in bilateral or contralateral lymph nodes

D.Distant metastases present
Ans B

 

following conditions is NOT a possible sequela of a tooth completely impacted in bone? 

Development of a dentigerous cyst around its crown.
B. External resorption of the tooth.
C. Osteonecrosis of the adjacent bone.
D. Development of a benign neoplasm adjacent to its crown

 

Psuedocyst: Aneurysmal bone cyst, traumatic bone cyst, stafne defect and mucocele

What cyst is a true cyst?
A- Dermoid

B- Stafne

C- Dentigerous

D- Nasolabial

Ans: B Aneurysmal bone cyst, traumatic bone cyst, stafne defect and mucocele NOT true

 

Which of the following is most likely to displace the adjacent teeth?

– Lateral periodontal cyst

– Dentigerous cyst

– Periapical cemental dysplasia

– Periapical abscess
Ans A

 

Baby with nodules on the palatal, what is it?
A) Bone nodules

B) Epstein pearls

C) Congenital epulis

D) Bohn nodule
Ans b

 

Neonate with numerous white nodules on alveolar ridge. What is it?

Eruption cyst

B) Bohn’s nodule
C) Congenital cyst of newborn
ans B

 

Epstein pearls: (keratin-filled cysts on midline raphe, not odonto)
Bohn’s nodule
(keratin-filled cysts rests of dental lamina odontogenic cysts)
congenital cyst/epulis of newborn: granular cell myoblastoma on gingiva

 

 

SALIVARY GLAND PATHOLOGY

Salivary flow hypofunction: unstimulated less than 0.1mL, and stimulated <0.7mL

 

Most common tumor OVERALL of salivary gland: Pleomorphic adenoma

Most commonly resembles parotid gland, mixed cell type, firm rubbery consistency

Most common tumor of MAJOR salivary parotid: Pleomorphic adenoma

Most common tumor overall of MINOR salivary: Pleomorphic adenoma

Most common MALIGANANCY of salivary gland: Mucoepidernoid, ACC

2nd Most common malignancy of MINOR salivary gland: PLGA

Adenoid cystic carcinoma ACC: Perineural invasion seen, cribriform, swiss cheese, highly reccurent, 15 year survival 10% (lethal), palate

 

Warthin tumor (Papillary Cystadenoma-lymphomatosum) in: 2nd benign parotid, oncocyte+lymphoid stroma

PLGA: second most common of minor malignancy
sialolith found in: Wharton duct (submandibular) need occlusal x-ray to dx

 

125- perineural invasion is seen in:

A.Adenoid cystic carcinoma(malignant palate)
B. Acinic cell adenocarcinoma (malignanat in parotid)

Mucoepidermoid carcinoma (malignant parotid)
Ans A

Bilateral parotid enlargement in

Silothiasis

Pleomorphic adenoma

I can’t recall it

Wharton tumor

 

Tumors of the salivary glands are:

1)uncommon and represent 2-4% of head and neck neoplasms

2)common and represent 75-80% of head and neck neoplasms

3)uncommon and represent 25-30% of head and neck neoplasms

4)common and represent 95-98% of head and neck neoplasms
Ans 1

 

Mixed salivary tumor:

1.Adenoid cystic carcinoma

2.High grade mucoepidermoid

3.PLGA

4.Pleomorphic adenoma
Ans 4

 

ORAL PATHOLOGY:

Found Presence of Supernumerary teeth in: Cleidocranial dysplasia

Anodontia/Oligodontia mostly related to: Ectodermal dysplasia (mostly seen in Alveolar bone)

Odontomas mostly associated with: Gardeners syndrome

Dens invaginates is commonly seen: Max lateral

Dens-in-dente most common in: MX LI
Most common site of osteo-fibrosis (Cementoma): Mandibular Anterior

Diagnosis of OKC: Histology

Hypercementosis, most common in PMs in Paget’s disease

Discolored of teeth seen: (Porphyria: purplish brown)(Cystic fibrosis: yellowish brown)(

Erythroblastic fetalis: blue greenish)

Erythroblastosis fetalis: Ring like enamel hypoplasia

Blue sclera is seen in: osteogenesis imperfecta, hypophosphatasia

Actinomyces oral manifestation: Lumpy jaw, sulfur granules

1st sign of multiple myeloma: Bone pain (“punched out” lesion in X-ray)

Osteosarcoma: Sun burst and uniform/symmetrical widening PDL, paresthesia

(numbness/tingling)

Scleroderma: widening of PDL + microstomia

Most common benign tumor in oral cavity: Fibroma

Lesion of alveolar ridge in infant: Bohn’s nodule

Dry socket is a form of Periostitis, Pathophysiology of dry socket: Fibrinolysis
Keratosis on the oral mucosa and dysplasia are more common in: tongue
Oral Hairy Leukoplakia: not premalignant, caused by EBV

When to do biopsy: 15 days after

Osteonecrosis most common with IV drugs: Zolmeda (zoledronic acid) & Aredia (Palmidronate), not with Oral (Fosamax or Boniva)

Nikolsky Sign: Pemphigus vulgaris (Acantholysis present) & Erythema multiforme

Subepithelial vesicular disease: Pemphigoid & Lichen planus

Chronic Desquamative gingivitis AKA Cicatrical pemphigoid

Cauliflower like pebbly appearance: Verrucous carcinoma, Condyloma accuminatum, Papilloma

Granular Cell Tumor (skin or mucosal tumor) histologically resembles: Congenital epulis, SCC

Lesion that resembles to SCC, but disappears in 16 weeks: Kerato-acanthoma (Skin tumor)

Most common malignancy found in: Metastatic Ca (Bone), Basal cell ca (skin), SCC/Epidermoid ca (oral cavity) Muco-epidermoid Ca (Salivary gland)

Swelling on maxillary lateral incisor area, doesn’t appear on rx. Histology: Pseudostratified squamous epithelium cystic lining. Diagnosis? Nasolabial cyst

Burning mouth syndrome (in postmenopausal woman) dry sticky bad taste, treatment: Capsaicin is the answer

Hypercementosis seen in: paget’s
First symptom of multiple myeloma: **bone pain
leukemia in children: ALL
feature of fetal alcohol syndrome: cleft lip s.s

Carcinoma of the tongue initially present as:
A. nodules on bilateral submandibular region.
B. nodule on unilateral submandibular region.
C. palpable regional lympnodes;

Necrotic ulcer on lateral surface of the tongue

Necrotic ulcer on lateral surface of tongue extends to floor of mouth

Ans D

 

1 Leukaemia gingival enlargement is seen only in dentulous patients.

2 Leukaemia gingival enlargement is seen in chronic leukemia.

A) both 1 and 2 correct

b) both 1 and 2 incorrect

C) 1 is correct and 2 is incorrect

D) 1 is incorrect and 2 is correct.
ans: C (1 correct, only seen in acute cases, NOT in chronic leukemia)

 

Which of the following statements are correct?

1 Chediak higashi syndrome Is primary neutrophil disorder

2 Down syndrome is not a secondary neutrophil disorder

3 lazy leukocyte is a primary neutrophil disorder

4 pepillion lefevre syndrome is primary neutrophil disorder

5 inflammatory bowel disease is secondary neutrophil disorder

A – 1,2,3

B – 1,3,5

C – 2,4,5

D – 2,3,4

Ans B

 

Pt have pain in one side of face this pain worse when moving his face what is most probably?

unilateral cavernous sinus thrombosis

Sinusitis

Acute periapical Abscess
Ans B (to tx: give: AB amoxi and nasal decong)

 

You did exfoliative biopsy and came positive with dysplasic cells what do you do next:

Confirm with another exfoliative biopsy

Incisional biopsy

Excisional biopsy

Wait

Ans B (need to always confirm exfoliative/cytology biopsy, it is an adjunct)

 

40 year-old man coming regularly to this office since 20 years, every 4-6 months for regular check-up. He has red/blue cyst on lower buccal side filled with mucous. Tx:

a.don’t worry – it’s viral infection

b.antibiotics

c.incision biopsy

d.excision biopsy

cytology
ans D

 

Patient has a large cyst. Appears radiolucent on radiograph. What to do first?

a.Aspiration biopsy

Excisional biopsy.

c.Incisional biopsy.

Antibiotics.

Ans A

 

What is not a characteristic of DI:

Short roots that breaks easy

Abnormal lines at the DEJ

Excessive deposition of dentin
ans: B. butttt if were talking about DI III then C is wrong, if talking about DI II (most common type) all above are correct b/c absence of scalloping at DEJ makes enamel chip off . check this

 

Case about kid with histopatologic biopsy result of Neuromas. Pick possible disorder:

Multiple Endocrine Neoplasia IIb

GardnerSyndrome

Cohn Syndrome

ans: MEN multiple endocrine neoplasia IIB (AD): multiple neuromas in lips, tongue, palate + medullary thyroid carcinoma+ pheochromocytoma of adrenal gland

 

Fordyce granules:

sweat granules

ectopic sebaceous granules

 

Which of the following is seen with hyperplastic (or was it associated with) foliate papilla:

A hairy tongue

B Lingual tonsil hyperplasia

C median rhomboid glossitis

D lymphadenopathy

 

 

Hyperplastic lingual tonsils may resemble which of the following?
a. Epulis fissuratum.

Lingual varicosities.

c. Squamous cell carcinoma

Median rhomboid glossitis.

Prominent fungiform papillae. (foliate papillae, not fungiform papillae)

 

Which of these lesions has the best prognosis?

squamous cell carcinoma on lower lip

melanoma on gingiva

adenocarcinoma on hard palate (PLGA)

other on lateral tongue or floor of the mouth

ans: adenocarcinoma PLGA

 

Which of the following has been most strongly implicated in the cause of aphthous stomatitis?

A- Cytomegalovirus
B- Allergy to tomatoes
C- Herpes simplex virus
D- Staphylococcal organisms
E- Human leukocyte antigens

Ans E

Lesion of basement membrane without acantholysis:

1) Pemphigoid

2) pemphigus

3) erosion lichen planus

Ans A

Why is important to rule out okc

a) due to potential for malignancy

b) asso with nevoid basal cell carcinoma

c) recurrence rate

d) infection

A, B , C correct, C most important

 

What is the first finding in HIV?

Decrease Hb

Decrease WBC

Kaposi sarcoma

B cell lymphoma

Ans B (leukoplaquia and candida seen before decrease in wbc, cd4 decreases after a while) kaposi seen in AIDs

116. An inherited disorder that presents as micrognathia and retrognathia of the mandible, glossoptosis and cleft palate. Pierre Robin Syndrome

cysts don’t occur in midline: cervical lymphoepithelial/branchial cyst and cystic hygroma coli

201- What doesn’t occur in midline?

A.Cleft lip

B.Lingual thyroid

C.torus paltinus

D.Nasopalatine duct cyst

Answer is A

 

1- Not need x-ray for diagnosis

Hutchinson incisor

Dens in dent

3 concrescence

4 dileceration

Ans 1

 

3- Most common site for sialolithiasis

1 submandibular gland

2 sublingual gland

3 parotid gland

4 minor salivary gland

Ans 1. To DX take occlusal Xray

 

4-Most common duct for sialolithiasis

1 stensen’s duct

2 bartholin duct

3 wharton’s duct

4 von ebren duct

Ans (3)

14- Which of the following associated with presence of supernumerary teeth

1 cleidocranial dysplasia

2 down syndrome

3 gardener syndrome

4 ectodermal dysplasia

Ans 1

 

15-Oligodontia mostly related to

1 trisomy 21

2 cleidocranial dysplasia

3 ectodermal dysplasia

Ans 3

Ectodermal dysplasia seen most in:
A- maxilla
B- mandible
C- alveolar bone

Ans C

 

16-Odontomas mostly associated with

1 neurofibromatosis

2 gardener’s syndrome

3 albert syndrome

Ans 2

 

17- what is the percentage of calcific metamorphosis in population

5-18 %

20-35 %

35-49%

4.50-70%

Ans 1

Origin of turner incisor?

Syphilis

trauma during delivery

trauma during pregnancy

trauma and infection

Ans. D

41-Which of the following Is Pre Malignant

1 odontoma

2 pagets disease

3 hairy tongue

Ans 2 (pagets can lead to osteosarcoma)

Proliferative verrucous leukoplakia associated to:
a. EBV
b. papilloma virus
c. others
ans b

75-Lumpy jaw appearance ( and some more information i can’t recall ) which disease

1 actinomycosis

2 cherubism

3 pagets disease

Ans 1

Patient came with a burning sensation. It happens mostly while eating food. Clinical examination, red lesion with white border on tongue and patient stated that it always moves around. What is the condition patients have?
A. Erythema multiforme
B. Erythema migrans
C. Erythroplakia
D. Ulcer
ans B. migratory, geographic tongue

Patient came to visit for checkup. He is currently taking both subepidermal nitroglycerine sublingually 3-4 times daily. Couple of months ago he used to take nitroglycerine weekly. What is the patient health status?
A. ASA 1 (healthy)
B. ASA 2 (mild, stable)
C. ASA 3 (mild, unstable, uncontrolled)
D. ASA 4 (Uncontrolled, unstable)
Ans D

24- 22 years old girl with fever, malaise, lymphadenopathy, multiple ulcers on tongue, palate with swollen gingiva

1 ANUG (anug necrotic ulcerative lesions on interproximal papilla ONLY)

2 acute herpetic gingivostomatitis (only on keratinized tissue)
3 marginal gingivitis

4 herpangina (on non keratinized tissue)

Ans 2

 

27-Patient girl has Pain when she turn her head to the right and swallow

1 lateral thyroid cyst

2 elongated styloid process

Ans 2

 

30-Tb oral ulcer present as

1 painless ulcer for long duration

2 mostly palate

Ans 1

 

Children with Fetal Alcohol Syndrome presents with what sign?

a) anecephaly

b) Midface deficiency

c) Cleft lip

b) Down syndrome

ans. B

66-dens invaginatus commonly seen in

1 max laterals

2 man premolars 3 max canine

4 man molars

Ans 1

 

70-How many population have Herpes virus in there bodies

35- 40%

50–55%

3.80-85 %

4.90-95%

Ans 3

 

lingual varicosis associated with

1 hypertension

2 gender

3 DM

4 xerostomia
5 age

Ans 5 (HTA and age are assoc)

106 – Most common deep fungal disease in USA

1 blastomycosis,

2 candidiasis,

3 histoplasmosis,

4 mucormycosis

Ans 3 (most common 2 s.s)

Conditions least likely to have alveolar bone loss in primary dentition

Hypophosphatasia

Leukemia

Poor Oral Hygiene

ANS B?

 

113 – trisomy 21 have all the following except

1 rampant caries

2 macroglosia

3 short roots

4 mandibular protrusion

Ans 1

acrylic mostly associated with candidiasis:

1 heat cured

2 chemical cured

3 light cured and one more option

ans 2 (least is heat cure it has less residual monomer, less porous and thus stronger and more colour stable)

 

82- Most important to determine any disease

1 history and clinical examination

2 blood test

3 biopsy

4 radiographic examination

Ans 1

 

83-Cleft palate / cleft lip mostly associated with which disorders

1 autosomal recessive

2 autosomal dominant

3 Hereditary

Genetically multifactorial
5. X-recesive

ans. 4 all of the above

 

79-Cleft lip is formed in which weeks during pregnancy

2-3

6-9

10-12

13-15
Ans 2

 

ANATOMY

 

pterygomandibular raphe: junction of buccinators and superior pharyngeal constrictor

Dental Lamina form: 6-7 weeks of utero
TMJ:
ginglimoarthrodial
rotation/terminal hinge axis of TMJ:
lower compartment (first 20mm)

Translation/sliding/gliding TMJ: upper compartment (next 30mm) when lat pterygoid simultaneously contract, disc and condyl move down the art eminence

Max mandibular opening: 50-60mm
max mandiublar lateral, max protrusive: 10mm

Max mandibular retrusive: 1mm

 

Facial nerve is most likely to be affected by cut or damage in all except which?

Internal acoustic meatus (7,8)

Jugular foramen (9,10,11)

Stylomastoid foramen (7)
D. Parotid gland surgery (7 passes in it so do retro V, superficial temporal A and maxillary A)
Ans B

After blodd clot formation what happens to healing

Increase osteoblastic

Decrease osteoblastic

Increase osteoclasts

Decrease osteoclasts

(callus formation then remodeling)

 

7-From anterior to posterior which order is correct

1 Inferior alveolar nerve, Inferior alveolar artery, lingual artery

2 lingual artery , Inferior alveolar nerve , Inferior alveolar artery

3 Inferior alveolar artery , Inferior alveolar nerve , lingual artery

4 lingual artery , Inferior alveolar artery , Inferior alveolar nerve

Ans (1) lingual nerve, ian nerve, ian artery, lingual artery

 

All are innervated by the Hypoglossal nerve, except?

Hyoglossus

B. Palatoglossus

Styloglossus

Genioglossus

Ans B

 

Which muscle is pierced during IAN block: Buccinator (if medial pt. cause Trismus)

 

Lymphatic drainage of The tip of tongue drains to:
a. submental

Deep cervical facials

Submandibular

Ans A

 

3) Which of the following anatomical landmark is used as indicator of the posterior border of ramus of mandible in inferior alveolar nerve block:
A. Mandibular foramen

Occlusal plane of mandibular posterior teeth

Pterygomandibular raphe (vertical portion)

Pterygomandibular raphe (horizontal portion) E. Coronoid notch

Ans A (if anterior boarder ans E)

 

28-Muscle to elevated tongue

1 styloglossus and palatoglossus

2 genioglossus and styloglossus

3 hyoglossus and palatoglossus

4 hyoglossus and genioglossus

Ans 1

muscle is most likely to get pierced if IANB is placed MEDIAL to the pterygoman raphe?

1 Buccinator and Lateral Pterygoid

2 Buccinator and Medial Pterygoid

3 Buccinator with Superior Constrictor Muscle

Ans 2 I think SCP and medial pterigoid

 

Posterior Superior Alveolar artery is a branch of?

maxillary from Internal Carotid Artery

maxillary from External Carotid Artery

ans B

 

The anterior loop of IAN can be predicted whe renerve comes: 

1 above mental foramen

anterior to mental foramen

below mental foramen

4.same level with mental foramen

ans 2

 

60-lingual artery direct branch from

1 ECA

2 maxilary A

3 IAN A

Ans 1

 

Highest chance of 2 canals in which mandibular teeth? (lowest in MX CI)

lateral

canine

1pm

2pm

Ans A.

 

If you cut or damage near the mylohyoid ridge, what will be affected?

Facial Nerve

B. Trigeminal nerve

Hypoglossal nerve

Glossopharyngeal nerve

Ans B

 

PHARMA MECHANISM OF ACTION:

MOA of Tetracycline: Bacteriostatic

MOA of Doxycycline: inhibit 30s ribosome/inhibit collagenous (inhibits MMP-8) longest duration tetracycline

MOA of Montelukast: Inhibition of leukotrienes (Used for Asthma and seasonal allergies)

MOA of Ranitidine: Reduce gastric secretion

MOA of H-antagonist (Antihistamine): Blocking histamine at the receptor

MOA of Ibuprofen: Reversible

MOA of Aspirin: Irreversibly inhibit platelet aggregation, inhibit platelet cyclooxygenase by

blocking the formation of Thromboxane A2.

MOA of Clopidogrel (Plavix): Alter platelet function, inhibit platelet aggregation irreversibly (Give pt allergic to Aspirin, no ulcer side effect, given to pt with past ulcer history)

MOA of Anti-depressant drug:

MOA of Warfarin: Inhibit vitamin K reductase resulting in depletion of reduced form of vit K,

Decrease K+ needed to synthesize factors II, VII, IX, X

MOA of Heparin: Anti-coagulant reversibly to anti-thrombin II & prevent conversion of

fibrinogen to fibrin

MOA of Dicoumarol: Anti-coagulant that inhibits vitamin K reductase & affects K-dependent

coagulation factors (Tx: Coronary Infarct/MI)
MOA Benzodiazepines: modulate activity of inhibitory NT (GABA) at the GABAA receptor

MOA of Xanax: Increase frequency of chloride channels on GABA receptor

Morphine relieves pain by: Acting on opioid mu receptor on neural cell membrane

MOA of Anticholinergic: inhibit binding of acetylcholine to muscarinic and nicotinic receptors

(found in eyes, secretory glands, nerve endings to smooth muscle cells)

MOA of Acetaminophen: Antipyretic effect by acting on centers in hypothalamus

MOA of Levodopa: Replenish deficiency of dopamine in patients with Parkinson’s

MOA of Reserpine: stabilize the axon terminal membrane preventing release norepinephrine

(Used for HTN)

MOA of Naloxone: Non-selective and competitive opioid receptor antagonist in case of opioid

overdose

MOA of Clonidine: Centrally acting sympatholytic (alpha adrenoceptor agonist)

MOA of Zoloft: Sertraline – Selective serotonin reuptake inhibitors

MOA of Sulfonylurea: Increased insulin production and sensitivity by Beta cells stimulation by

binding to ATP dependent K channel/ Stimulation of pancreatic beta cells to secrete insulin.

MOA of Sulfonamides: Inhibit folic acid suynthesis

MOA of Bisphosphonate: Inhibit the Osteoclast via apoptosis

MOA of Periostat: Inhibit collagenase/protein synthesis

 

 

PHARMA CONTRAINDICATION:

Acetaminophen is contraindicated with Liver disease (cause hepatotoxicity) and alcohol

Nitrous Oxide contraindications: 1st trimester only, nasal congestion, COPD

Asthmatic pt: Used Acetaminophen (Tylenol), Avoid using Aspirin, can cause Hyperventilation( asthma attack), Macrolides avoided in Asthma (interact with Theophylline)

Antihistamine is contraindicated with Erythromycin: Terfenadine

Epinephrine (Adrenalin) should NOT be used with tricyclic anti-depressant, hyperthyroidism
max epi given in cardiac pt: 0.04 mg (2 carpules of 1:100,000 and 4 carpule of 1: 200,000)

In multiple sclerosis: LA with epi is contraindicated

Erythromycin and Tetracycline are prescribed carefully in patients with peptic ulcer because:

interact with Antacids

Penicillin is cross allergenic with Cephalosporin because of Beta Lactamase
Never give penicillin with tetracycline: due to Antagonists property

Chloramphenicol contraindicated in G6PD, Side effect of chloramphenicol: Aplastic anemia

Strongest glucocorticoids: Dexamethasone

Benzodiazepine antagonist: Flumazenil, Opioid antagonist: Naloxone

Detoxification of Morphine addiction: methadone

Mixed agonist-antagonist analgesics: pentazosin, nalbuphine

Use for sedation on child: (Don’t use Meperidine)
Antibiotic for non-odontogenic maxillary sinusitis: Augmentin (Amoxicillin + clavulanic acid)

ANUG with lymphadenopathy: Metronidazole (s.s)

Med for Trigeminal Neuralgia: Anticonvulsive drug, carbamazepine

Child Used Amphetamine med: ADHD (if kid takes, tell him not to take prior dental app)

Heparin is contraindicated to pt with taking gingko biloba

Aspirin is contraindicated with Coumadin

Ginseng is contraindicated with Aspirin, Warfarin, NSAIDs

Glucocorticoids are contraindicated in Diabetes

Benzodiazepines (Diazepam) is contraindicated for pregnant women, myasthenia gravis, acute

narrow glaucoma, COPD, emphysema.

cholinergomimetics are contraindicated in:

Allergic to Aspirin: Take Acetaminophen, not Ibuprofen

Pregnant Women: Use Acetaminophen, No NSAIDs (causes patent DA)
Antibiotic used in gingival cervical fluid for periodontal bacteria: Doxycycline,

HIV related oropharyngeal candidiasis: Systemic Fluconazole

Antifungal for TROCHES: Clotrimazole

Most common medicine for Grand mal: Dilantin

Ethosuximide is most commonly used: Absence epilepsy (Petit mal)

Diazepam for status epilepticus

Drugs caused gingival hyperplasia: Phenytoin, nifedipine, cyclosporine

Meds cause Xerostomia: Diuretics, CCBs, Antihistamine, antimuscarinics (glycopyrulate/scopolamine)
Tx for xerostomia with Sjogren’s syndrome: Cevimeline HCL, pilocarpine (20 mg a day 5q4h)

Coronary infarct: Dicumarol (Vit K antagonist)

Antidote of Warfarin: Vit K

Antidote of Heparin: Protamine sulfate

The most potent & most toxic LA: Dibucaine

Long acting LA with less toxic: Bupivacaine

Pain medication for liver toxicity & renal toxicity: Oxycodone

Tx of Myasthenia gravis: Physostigmine & Pyridostigmine

Tricyclic Antidepressant: Imipramine & amitriptyline

Best initial tx for TMJ muscle spasm: NSAIDS + Muscle relaxant

Tx for Rheumatic arthritis: Adalimumab & infliximab (bind to TNF – alpha receptors)

Percocet is class II schedule drug (Oxycodone + Acetaminophen)

Motion sickness: Scopolamine, Diphenhydramine (Benadryl)

Drug choice for pt with bradycardia: Atropine

Asthma: Albuterol (short acting) and theophylline (long acting)

Amantadine antiviral drug used for: Influenza A & Parkinson Disease

Epi to reverse give: Prazosin or chlorpromazine

Serotonin syndrome: Tramadol (narcotic pain killer like morphine) + SSRI
Tx of Parkinson disease: Carbidopa + Levodopa

Pt with sleep apnea, which pain med should give: Ibuprofen/acetaminophen

Tx of lidocaine induced seizure: Diazepam

Tx of Depression & Anxiety for Obsessive Compulsive Disorder: SSRIs

Best benzodiazepine for pt with liver cirrhosis: LOT (Lorazepam, Oxazepam, Temazepam) due to not metabolized by liver, so safe for to not induce liver failure.

Best benzo for IV sedation: Midazolam

Pt allergic to ester & amides: Use Diphenhydramine (Benadryl)

Laryngospasm: Tx with Pure oxygen, if persistent use Succinylcholine, if severe then

cricothyroidism

Malignant Hyperthermia: tx with Dantrolene

ADHD Kid: Ritalin (methylphenidate hydrochloride)

For Hirsutism: Eflornithine

Adrenal Crisis: Cardiac shock, hypotension, CV collapse, 100mg 0.9% hydrocortisone with saline (if minor qx, pt taking 15mg of prednisone, or 5 mg of albuterol inhaler daily, of ext give 50 day of and 50 after, for multiple ext give 100 day before 100 day after. If taking very high dose of cortisol but only for 3 days not needed, just if its more than 2 weeks of 15 or more or 2 months of 5mg or more)

Adrenal insufficiency: caused by prolonged regimen of Corticosteroids, more than 2 years

Thyroid Crisis: Hypertension and Increased HR

Anti-anxiety in pregnant women: Promethazine

Most common cause of xerostomia: DRUGS
Tx of xerostomia: pilocarpine Civemeline HCl

drug-induced gingival hyperplasia: phenytoin, niphedipine, cyclosporine

What drug use to reverse meperidine(Demerol) effect? Naloxone
Best drug to reverse effect of Benzodiazepine: Flumazenil (benzodiazepine antagonist)

142- day 2, antibiotics for ANUG? metro (s.s)

143- day2: all true for transient ischemic attach except; give nitroglycerin

146 – antibiotics that is safe for renal hemodialysis: I put clindamycin

147- antibiotics for mycoplasma: erythromycin, clarithro, tetra

150- patient allergic to ampicillin what to give him: azithromycin 500 mg/ clinda

163- aspirin 12 mg/ml in plasma concentration how ml after three half lives? Answer: is 1.5

12–> 6 —>3 —>1.5

PHARMACOLOGY

 

Best drug to reverse effect of Midazolam (Versed): Flumazenil

Glucocorticoides medication can cause : cushing syndrome (thining of skin (strae), moon facies, adrenal crisis in Ux, central adipose, muscle thinning, infections, immunosupression)
opioid causes all except? a.resp depression, b. pinpoint pupils, c. peripheral pain, d. somnolence ans c

the metabolites of codeine are: morphine and hydrocodone

metabolites of morphine: hydromorphone, normorphone

action of ranitidine? inhibit gastric acidity

question about betel quil ..s.s

 

Antibiotic Prophylaxis for:

a) Mitral valve with regurgitation

b) Implanted cardio stent

c) Valve replacement

d) Pace maker

ans C

Test to do for patient taking Coumandin a. PT/INR (test for people taking heparin? PTT)

RQ- 1 dose of aspirin can cause bleeding up to:

A- 4 hours

B- 10 hours

C- 1 week

D- 1 month

Ans C

Warfarin mOA: Warfarin binds to vitamin K epoxide reductase complex subunit 1 and irreversibly inhibits the enzyme thereby stopping the recycling of vitamin K
warfarin: teratogen

nitroglycerin side effects (vasodilation/ SE headache syncope tachy methemoglobulenemia : causes chocolate blood and cyanosis of lips)
assoc with methamoglobulenemia: Benzocaine, Chloroquine, Ciprofloxacin, Dapsone, Flutamide, Isosorbide dinitrate, Metoclopramide, Naphthalene, Nitrofurantoin, Nitroglycerin, Nitric oxide, Phenazopyridine, Phenelzine, Phenobarbital, Prilocaine, Primaquine, Quinine sulfate, Sulfonamides, Trimethoprim

172- patient child use amphetamine what he have : ADHD

Patient use Anticonvulsants drug what he have: Trigminal neuralgia

 

 

.Which medication to control excess of saliva on an IV patient with ketamine administration: Glycopyrrolate

Which of those medications doesn’t match with the disease?
a. Ganciclovir…. CMV
b. Zidovudine with chickenpox
c. Adamantine with influenza A
ans b. for HIV

10- NSAID therapeutic effect

1 reducing the production of prostaglandins

2 Cox inhibitors

3 increase the production of prostaglandins

Ans 2

 

42 -pseudomembranous colitis caused by clindamycin is

1 overgrowth of C. diff

2 treated by IV metronidazol

Ans both are true. C.diff treated with metro and vanco

 

The platelet-aggregation blocking effect of aspirin is reversed only by the:

NaHCO3 (asprin toxicity)

Vit. K (warfarin)

Protamine (heparin)

Platelets (asprin)

Hemodialysis (everything)
ans D

 

Pt with myasthenia gravis, which antibiotics can you give:

Erythro

Clarithro

Imepenem

Penicillin

Ans: D

 

Treatment of glaucoma:

Pilocarpine

Betaxolol

Latanoprost

Bimatoprost
brimodine

All of the above

Ans: all of above (diazepam is contraindicated, same with any muscarinic blockers)

 

 

13-The drug-receptor activity of naloxone is best characterized by which of the following pairs

1 High affinity, No intrinsic activity

2 Low affinity, High intrinsic activity

3 No affinity, High intrinsic activity

4 No affinity, Low intrinsic activity

5 High affinity, High intrinsic activity

Ans 1

 

Burning mouth syndrome treatment: Capsaicin or antidepresants

anti tartar in tooth paste: tetrasodium pyrophosphate
desensitizing: potassium nitrite
antiplaque: tricoslan

 

63- ginseng contraindicated with

1 Aspirin

2 Penicillin

3 Clindamycine

Ans 1

 

73- Antidepressant drugs mode of action

1 blockage of amine reuptake

2 Selective serotonin reuptake inhibitors

Both are correct depending on what were talking about SNRI, SSRI, MAOI

 

74-patient has signs of opioid toxicity what u should give

1 aspirin

2Naloxone

Ans 2

 

 

92 – How many cartridge of LA 2% lidocaine u can give to child 45 lbs

1

3

6

9

Ans 2 (3 cartridge )

 

Fastest acting LA?

A – lidocaine

B- Articaine

Ans. B

 

119- Angioneurotic oedema is mostly occurs with which of the following LA

A.Articaine

B.Lignocaine

C.prilocaine

D.mepivecaine E.Bupivecaine

Ans A

 

Most potent LA? Bupivacaine

cause of methemoglobinemia: prilocaine, lidocaine, bupivacaine, topical benzocainse** (tx: IV methelyne blue)
toxicity of LA: IV injection or too much (biphasic response. Initially: tachy, HTA, confusion, tinnitus, metallic taste. Later: tremor, halluc, hypotention, brachC, seizure, resp/CV arrest)

 

NITROUS (blue cylinder)

Nitrous oxide edverse effects: nausea (diffusion hypoxia)

Contraindication of nitrous: 1st trimester prego (give promethazine), COPD, nasal congestion, collapsed lung, very nervous child, resp infection

If nitrous abused:oxidizes the cobalt in vitamin B12, resulting in the inhibition of methionine synthase, can lead to peripheral neuropathy

major advantage of using nitrous oxide?

Nitrous Disadvantage: vomiting, lack of potency

The correct total liter flow of nitrous oxide-oxygen is determined by the amount necessary to keep the reservoir bag: 1/3 to 2/3 full.

Nitrous oxide Total flow rate: 4-6 L per min

First sensation from N2O: tingling of fingers

Device used in evaluation of N20? Pulse oximeter

the least person to get exposed from toxic effect of nitrous oxide? Patient (most staff)

max nitrous given to children: 50%

max nitrous given to adults: 70% (stop switch)

All of the following can be used in dental phobia except:

.Nitrous oxide

Anti anxiety oral med

Sedatives drugs

Positive reinforcement

Tell show do technique

Relaxation

Ans TSD (not nitrous for phobic pts) s.s

 

A 26-month old child w/ 12 carious teeth. How to treat?

General Anesthesia

Oral sedation

Nitrous oxide

local anesthesia

Ans A

 

What true with concious sedation:

1.Patient should keep eyes open

2.Patient shouldn’t fall a sleep

3.Patient should have defensive response

4.Patient should have verbal communication completely or all the time

 

fail-safe mechanism on an analgesia machine for nitrous oxide and oxygen prevents the delivery of nitrous oxide greater than

A- 20 percent

B- 50 percent

C- 70 percent

D- 90 percent

Ans C

 

25 yo female breast feeding 12m old child and currently pregnant, which sedative ?

Halcion

Promethazine

Nitrous

Diazepam

Phenobarbital

 

A patient with mild sedative what do u expect regarding his response?

Can not verbally communicate

Maintain heart function

Doesn’t respond to touch

Loss all normal protective response
ans B

 

A patient with mild sedative what do u expect regarding his response?

Can maintain his airway independently

Can’t maintain his airy

Can’t communicate verbally
Ans A
Stage 1 of geudels stages of general anesthesia (amnesia and analgesia) is conscious sedation *best way to monitod is verbal response* (3 signs to indicate correct level of sedation reached verrills sign, slurring speech, burring vision)

 

ORAL SURGERY:

Minimum platelet count for oral surgery: 50,000 (normal 150k-400k)

Most common facial fracture nasal bone fracture, 2nd most mandibular fracture

Mandibular fracture: most common condyle, angle, then body

Most common mid face fracture: Zygomatico-maxillary

Alveolar fracture, closed reduction: semi-rigid MMF/IMF splint for 4-6 weeks unless (have callus formation unlike with rigid internal fixation RIF, no callous)

condyle fracture: open/close reduction + IMF max 2 weeks or get ankylosis

after BSSO have internal fixation, but use occlusal splint for is 4-6 weeks

Paresthesia in lower lip: Angle of mandibular fracture, after EXT, malignancy

Lefort I fracture Guerin Sign (transverse maxillar): crepitus and ecchyosis in buccal mucosa and greater palatine vessels region, (Lower midface, floating palate), open bite

Lefort II fracture (pyramidal): periorbital edema, paresthesia of infraorbital nerve, Subconjunctival hemorrhage

Lefort III fracture (naso-ethmoido-frontal complex, cranio-facial): same as above+ Subconjunctival hemorrhage+ Rhinorrhea ([CSF] leak into nasal cavity)

zygomatico-maxilary: flattening on cheek bones, same as above + May have limited mouth opening if interfere with coronoid process

blow out fracture “orbital fracture”: results in diplopia, eye looks down

Bilateral Sagittal Split Osteotomy (BSSO): correct malocclusion of mandible, mandibular

retraction, mandibular augmentation or asymmetry. Worse complication: IAN paresthesia neurosensory loss (mostly to pull fwd but also back)*condyle not moves

vertical ramus: specifically to put back**

Distraction Osteogenesis (DO): done in ant mandible, lengthen only not widen. Vs. BSSO DO done in young age, growth seen after, less nerve damage b/c 1mm activation per day, less time, less relapse, but more discomfort and more follow up

Correct bimaxillary Class III: Le Fort I + BSSO

Worst place to do graft: Canine eminence, interdental

Submandibular drain into what space: Deep cervical lymph nodes

OS instruments: Forceps and elevators:

Maxillary:

# 150: universal maxillay

#286: max root tips

#65: Bayonet: max roots

#88: L/R molars, cowhorn

Mandibular

151: ant mandible, A: PM

cryer elevator: Mand root

#23: L/R molars, cownhorn

#23 and 222 molars

#65 forceps, usually used for removing root tip

Elevator acts as: Levers, engage below Alveolar crest

One side tissue suture: Interrupted (immobilize the flap, 2-3mm apart, 2-3 from free edge, from movable to non-movable, disadvantage takes more time)

Only do continuous on vermillion border

Most to Least Frequent Impacted teeth: Mn M3, Mx M3, Mx C

Easiest to extract Mn M3: Mesio-ang, horizontal, vertical, Disto-ang(opposite for MX)

Ideal time to remove when M3 root is 2/3 formed

During extraction, which direction tooth should luxate: Child: Palatally, Adult: Buccally

Most sever complication EXT: fracture of tuberosity (maxilla), IAN injury (mandible)

Tuberosity: fully ext, smooth boone, if attached mobilize w suture

Most common complication of EXT: Bleeding (maxilla), infection, Trismus, *dry socket (mandible), root fracture (during qx)

Causes of Alveolar Osteitis (Dry Socket) cause: Active dislodgement of blood clot (Fibrinolysis of the Clot, usually around day 3) smokers/oral contraceptive

Symptom: trobbing pain, fetid odur, bad taste

Tx: Irrigation with Sterile solution & Medicinal/Sedative dressing every 48 hr, to control pain: Analgesics. NO ANTIBIOTICS NEEDED, NO curettage

EXT M3 and root displacement Mx: infratemporal space, Mn: submandibular space

Caudwell Luc Technique: removal of root tip from Max sinus, incision over canine fossa, we souldnt do it tho only oral surgeron

Biggest risk with EXT of a lone single (ankylosed) remaining MX M: Fracturing tuberosity/fracturing ramus

What order do you extract upper posterior molars & why? Order of extraction of teeth in maxillary molars: M3, M2,M1 to prevent fracture of tuberosity (max before mandible and most posterior teeth first)

Non-rigid splint is recommended for Subluxation, Luxation, Avulsion to avoid Ankylosis

MADS blood supply to TMJ: maxillary, ascending, deep auricular, superficial temporal

TMJ surgery there is risk of damage to: facial N

Freys syndrome: auriculotemporal N damage (gustatory sweating, after parotidectomy)

Type of X-rays to see Fractures:

Pano: best for mandible fracture

Reverse towne: for condyle fracture

SMV: zygomatic fracture

Water’s: for maxillary sinus

CT: facial best of all***

Sympisis: AP/ occlusal (symphysis closes at 6-9 months)

primary consequence of trauma to jaw in kids: retards growth and asymmetry (key signs are occlusal discrepancy)

lefort I surgery: maxillary sinus, nasal widening

A 40 years old patient bilateral posterior crossbite. Tx? Surgery: Maxillary ostectomy

Cannot do rapid palatal expansion, this is only in child

Infection of Mx CI/C: canine space, MX PM: max sinus, MX M2: buccal space MX3: infra

Infection of Mn I: mental, Mn M3: submandibular or submassateric

risk of infections in the face region? Anterior triangle cavernous thrombosis, veins with no valves. (danger triangle from ophthalmic vein or facial) cavernous sinus thrombosis if canine space or deep temporal space is infected

for it to go to mediastinum: retropharyngeal

 

A displaced fracture of the mandible courses from the angle to the third molar. This fracture is potentially difficult to treat with a closed reduction because of

A- Injury to the neurovascular bundle

B- Malocclusion secondary to the injury

C- Compromise of the blood supply to the mandible

D- Distraction of the fracture segments by muscle pull

Ans D (unfavourable fracture)

 

Fracture of which part of the face would compromise pt’s respiration?

A) Fracture through the body of mandibular (bilateral)

B) Fracture to condyle

c) Fracture to angle of mand

bilateral mandible fractures may result in posterior displacement of the tongue resulting in airway obstruction.

 

ecchymosis in Floor of mouth after trauma:

Bilateral fracture of mand

Ranula

Body of mandible

Ans 3. Coleman’s sign (guerlin sign for  Lefort I)

 

Patient right eye is drooping, loss of sensation……what the possible area is fractured?

Infra-orbital border of eye

Maxillary process of sphenoid bone

Lateral border of maxillary sinus
Ans A: blow out fracture”

 

After, qx a mesial root tip on a molar extraction breaks, what’s the first thing you do?

get hemostasis and visualize the root

take an x-ray

pick at it with root pick

surgical retrieval
ans. A

Patient came to the office with Oro-antral fistula 6mm, 1 week after extraction.

Treatment: buccal flap (if communicate at time of surgery less than 2 nothing, if between 2-6: suture, if more than 6 flap)

 

 

proper placement of a lower universal extraction forceps for the removal of tooth 4.6 involves placing the beaks of the forceps

as far apically on the tooth root as possible and applying apical pressure during luxation.

on the lingual and buccal enamel of the crown and applying apical pressure during luxation.

at the cementoenamel junction of the tooth and gently pulling upward during luxation.

as far apically on the tooth root as possible and gently pulling upward during luxation.

at the cementoenamel junction of the tooth and applying a rotational force during luxation.

Ans A

Why smoking increases the rate of dry socket?

A) reduced capillaries permeability

B) reduced healing

C) vasoconstrictive effect on blood vessels

D) all

Ans C and B

 

Patient wants an implant, what the best INR level dentist should consider?

INR procedures:

EXT normal pt: 3.5

INR pre-op (<2.5)

o Aspirin: <100 mg/day: no change

o Aspirin: >100 mg/day: stop 5-7 days prior to surgery

o Plavix (Clopidogrel): talk to doct

o Coumadin (INR <2.5): no change

o Coumadin (2.5-4): physician consult, stop 2 days pre-op

o Coumadin (4<INR): physician consult, stop 2-5 days pre-op, and check

 

A healthy person has an ANC between 2,500 and 6,000. ANC of <1000 no tX

Which local infiltration you give for maxillary premolar

Endosseous Local Infiltration

Paralingamental

Transligamental

Supraligamental

Ans C (another name if innjected into PDL)

 

 

Amide anesthesia synthesize in plasma:

Articaine-septocaine
b. Azitromicyne

Prilocaine

Ans A

 

Surgical guide for all except:

Size

Angulation

Location

Number of implants

Ans D. number of implants will be given by CBCT or space available

 

After extraction of 8 what is the percentage of alveolar bone resorption?

1.10%

2.40%

3.80%

4.100%

ans 40

Resorption of bone takes place in which direction after extraction? Max: up and in

Mandible: down and out (s.s)

 

31- In genioplasty surgery there is risk of damage to , in retromolar flap lingual N

1 facial nerve

2 lingual nerve

3 mental nerve

4 hypoglossal nerve

Ans 3

 

 

Greatest risk to injure IA nerve on extraction of 3rd molars:

Lack of visualization of end of roots

Root tips sit on top of mandibular canal

Horizontal impaction
ans3

 

What could be least possible in differential diagnosis of unilateral pain over TMJ, headache , restricted mouth opening, 3 weeks after wisdom tooth extraction:

1-TMD

2-Neoplasm in mandibular molar region

3-Cellulitis

4-Infection of extracted socket

5-Hemicranial headac

Ans 2

 

The most appropriate time to remove a supernumerary tooth that is disturbing the eruption of a permanent tooth is

as soon as possible. (through palatal flap)

after ⅔ to ¾ of the permanent root has formed.

after the apex of the permanent root has completely formed.

after the crown appears calcified radiographically.

Ans A

how to make incision to remove mandibular tori? Intrasulcular incision if there’s teeth and least one tooth after for better accessibility, and if edentulous on alv ridge NO vertical releases needed (envelope)

 

In removing a torus palatinus, the practitioner inadvertently removed the midportion of the palatine process of the maxilla. One would expect to see

A- The nasolacrimal duct

B- The inferior nasal concha

C- An opening into the nasal cavity

D- An opening into the maxillary antrum

And C (indicated in chronic irritation, denture, speech interference, place palatal stent to prevent hematoma formation and to support flap. Double Y flap is required for palatal torus)

 

A stent for palatal flap for what?

A- Preserve flap displacement

B- To eat patient normal food after surgery

C- To improve the nutrition of the flap

D- Prevent the flap from thermal injury
Ans A

 

In which of the following mandibular fracture cases should the intermaxillary fixation be released earliest?

A- A high condylar fracture

B- A fracture through the site of an impacted third molar

C- An angle fracture and contralateral parasymphyseal fracture

D- A jaw in which the treatment has been delayed by the management of other injuries

Ans A (muscles that up fractured segment up in condylar fracture are sling muscles and temporalis meanwhile lateral pterygoid of contralateral moves TWDS side of facture +contralateral open bite)

 

Lefot 1 include which of the following

1 Max sinus
2 orbit
3 zygomatic process
Ans 1

 

Recurrent ranula treatment?

Marsupialization

Flap surgery

Removal of the ranula and part of sublingual gland

ans 3

 

after man 3rd molar extraction what may happen

1 parasthesia, trismus, infection

paresthesia, fracture, infection

3 parasthesia, bone fracture, alveolar ostitis

4 bleeding, fracture, trismus

Ans  1 (bleeding, infection, dry socket, Trismus)

 

 

Damage to the lingual nerve following a mandibular third molar extraction is

preventable in most cases.
B. not likely to recover
C. more frequent than damage to the inferior alveolar nerve
D. too rare to inform the patient during consent
Ans A (never sever section the tooth completely and never do trough in lingual corex)

 

You extracted a tooth and gave penicillin. Next day pt has high fever, and dysphagia. What do you do?

A)Add another drug
2) change the antibiotics
3) refer to OMFS

Ans: pt has Ludwig angina (submandibular, submental and sublingual spaces bilaterally) refer to OMFS for I&D

 

not involved in Ludwig’s angina?

1 Sublingual space

2 Submandibular space

3 Retropharyngeal space

4 Submental space

ans 3

 

Immediate Alveolar osteitis treatment:
a. Chlorhexidine rinse at home
b. Irrigation on the area and sedative dressing
c. Antibiotics
d. Curetaje of alveolus
ans B

 

Which of the following is the most likely cause of ankylosis of the TMJ

A- Neoplasm

B- Rheumatoid arthritis

C- Traumatic injury

D- Developmental abnormality
ANS: C cause is trauma, most common complication of  RA is ankylosis bilaterally

What do you use to evaluate a TMJ disc?

CT Scan.

PA

MRI (Answer). (radiowaves)

Antero-Posterior.

 

A patient has a skeletal deformity with a Class III malocclusion. This deformity is the result of a maxillary deficiency. The tx of choise is:

orthodontics.

surgical repositioning of the maxilla.

anterior maxillary osteotomy.

posterior maxillary osteotomy.

surgical repositioning of the mandible.

Ans B but if before age    can do reverse pull head gear

 

What true with concious sedation:

1.Patient should keep eyes open

2.Patient shouldn’t fall a sleep

3.Patient should have defensive response

4.Patient should have verbal communication completely or all the time
ans 4

 

BISPHOSPHONATES:

Oral bisphosphonates: Alendronate (Fosamax), Ibandronate (Boniva), Risedronate (Actonel)
IV: Pamidronate (Aredia), Zoledronic Acid (Reclast), Ibandronate (Boniva)
MOA bisphosphonates: inhibit osteoclasts via apoptosis
Bisphosphonates indications: osteoporosis, bone metastasis, pagets, multiple myeloma or cancer metastasis
Risk of BRONJ oral is increased as duration of oral bisphosphonate therapy exceeds: 3 years
Risk of BRONJ for IV: 2 months
if taking longer than 3 years with cortico: A 3 month drug holiday. surgeries and dental implant placement should be avoided, and endo treatment should be considered before extractions
osteoporosis bisphosphonates usually given usually orally
bisphosphonate IV for 2 years absolute contraindication: for implant, EXT, ortho
bisphos half life of 10 years or longer

 

Biophosphonate

1) are generally given in IV formulation for osteoporosis

2) have half life of 10 years or longer

3) at lest 12 weeks of exposure is one of the diagnostic criteria for BRONJ.

Ans (for dx criteria of BRONJ: 8 weeks/2 months atleast)

pt needs full mouth EXT for denture, taking zaldendronate, how to treat?

Do full mouth ext

Ext questionable teeth

Refer to specialist? maybe

RCT and de-coronate

Ans: D (never EXT, maybe specialist)

 

Patient has BRONJ and bone is exposed, what is treatment?

A) hyperbaric oxygen
B) sc/rp
C) chlorhexidine rinse and oral antibiotics
D) all
Ans: C but depends on the stage

Stage 0: no tx
stage 1: asym exposed necrotic bone tx: CLX
stage 2: sympt exposed bone with soft tissue infection tx: CLX, antibiotics

Stage 3: sym exposed bone, and fracture tx: CLX, antibiotics, debridement

 

Osteoradionecrosis and BRONJ:

have identical pathophysiology

demonstrate similar clinical presentation.

occur more frequently in the maxilla

can be prevented by hyperbaric oxygen therapy
ans B

 

OSTEORADIONECROSIS

Osteoradionecrosis is above: 60+grays, ORN occurs in mandible
cause of ORN
: hypocellularity, hypoxia, hypovascularity.
Osteoradionecrosis pt indicated in EXT: Use Hyperbaric O2 for angiogenesis but prefered tx is Endo and decrown too

What is the first sign of damage after acute irradiation

a- death

b-erythema

c-hair loss

Ans B

 

Treatment of Osteoradionecrosis:
A. Antibiotic coverage
B. Conservative treatment including antibiotic coverage and resection of jaw segment.
C. Conservative treatment with sequestrectomy
ans B

 

Osteoradionecrosis is mostly related to

seen in maxilla

seen in mandible

related to bisphosphonate use

happens when radiation is 42.5Gy

Ans B

 

A patient received radiation therapy and requires extraction, treatment be?
A)extraction with alveoloplasty and sutures
B)extraction with alveoloplasty of basal bone and suture
C) pre-extraction and post-extraction hyperbaric oxygen
D) no extraction
ans C

IMPLANT

 

need 8mm of bone height in mand, in maxilla (need 10mm in length of implant)
use CBCT for implant placement not pano (orthopantogram)

Implants- implant: 3mm, tooth-implant: 1.5

IAN- implant 2mm from IAN, implant-mental foramen 5mm

Implant max sinus: 1mm

high torque, low speed (max torque applied 35Ncm and max rmp of hand piece:15 rpm)

irrigation: with saline**

countersink implant: flaring or enlarging the coronal end of the osteotomy

implant/ficture level impression: the impression coping/post attached to the implant

abutment level: coping attached to abutment

During the first stage of implant surgery: the implant fixture is placed into the bone, and a cover screw is inserted. after healing, 2nd stage: is put healing abutment and test torque implant.

Healing period: Maxilla: 5- to 6-month Mandible: 3- to 4-month healing period

In single-stage procedure: the implant healing abutment would be placed immediately, and no cover screw would be utilized.

Larger diameter implants offer greater surface area for osseointegration, thus provide greater implant stability than height

Placement of an endosseous implant after grafted alveolar cleft should occur after a 4-month consolidation period.

In a 15 year old pateint, #8/9 fell out, can put implant b/c at age 12 vertical growth of maxillary bone complete.

If infection at EXT site must wait 5 months prior to implant placement

If heat above 47 degrees, primary stability osteointegration fails

Preload implant is comparable to: torque

Biting load of denture comp to real teeth is 1/6 or 1/5 (less)

implant analog definition: replica of entire dental implant, not intended for implantation rather used in lab to fabricate abutment in lab, to pout cast with

worst force to implants: horizontal

Best anchorage for primary stability: D1 because its cortical

Better vascularity and good primary anchorage: D2

Best bone for osteointegration? type 2 (D2)

Best prognosis for implant?Anterior mandible (has D1 and D2)

Worst prognosis for implant: post maxilla (D4 bone)

function of internal connection between implant and abutment? Antirotation (internal hex better, external hex unscrews)

most common sign of failure: mobility

max taper for o ring: 15

 

Minimum distance between 2 implants

1 .1mm

2mm

3mm

4mm

Ans 3

How much minimum buccolingual bone thickness needed for 4 mm implant

5mm

7 mm

9mm

11mm

(There was no 6 mm option) Ans 2

 

How much distance from inferior alveolar canal and implant should be

1mm

2mm

3mm

4mm

Ans 2 (from mental 5mm)

 

55- How much distance from maxillary sinus and implant should be

1mm

2mm

3mm

4mm

ans. 1 (from buccal/lingual plate, inf border, max sinus, nasal cavity) from natural tooth 1.5 (always add bone graft to sinus floor not to ridge)

 

Main reasons implants fail to integrate:

premature loading

apical migration of junctional epithelium

overheating during placement

loose fitting implants

smoker, alcohol

radiation

all of the above

ans 7 all of the above

 

Why you put implants instead of tissue born RPD in lower arch which is opposite upper natural dentation

1 to distribute the force

2 to decrease bone loss

3 to increase support

Ans 2

 

two implants to support denture

implant gives both support and retention

2.implant gives support, tissue gives retention

3 implant gives retention, tissue support

4 implant gives retention tissue retention

Ans 3 (4 implant min for maxilla, 2 for mandible)

 

 Platform switching

1) typically refers to larger diameter implant and smaller diameter abutment

2) requires less bone remodelling post abutment placement

3) decreases risk of screw loosening

Ans A and B are correct

 

Where should you put implant platform in esthetic area?

At level of alveolar crest

below opposing tooth gingiva

1mm subgingival to adjacent teeth CEJ

1-2mm above

ans 3 (should be 2-3 mm below CEJ of adj tooth)

Cervical position while placing an implant, how should the implant be placed in relation to adjacent CEJ? I chose 2-3 mm apical the adjacent CEJ

 

most common reason why implant fail?

Lack of primary stability

over heating

smoking

qx error

Does not osteointegrate
ans D (90% 10 year success rate of implants)

 

A fixed dental prosthesis that uses an osseointegrated implant as one abutment and a natural tooth as the other abutment is likely to fail due to which of the following?

(1)There is difficulty attaining proper esthetics.

(2)Obtaining a favorable path of insertion will be difficult.

(3)The implant and the natural tooth will exhibit different mobility.

Ans 3

 

Cervical position while placing an implant, how should the implant be placed in relation to adjacent CEJ: 2-3 mm apical the adjacent CEJ

When osseointegration occurs, which of the following best describes the implant–bone interface at the level of light microscopy following osseointegration?

Epithelial attachment

Direct contact

Connective tissue insertion

Cellular attachment
ans B. direct

 

in implant preparation, which of the following can be used?

A) hydroxyapatite irrigation

b) High-Speed Hand Piece

c) Low torque Drill

d) Saline Coolant

ans D

 

least likely to cause problems in OH:

Open contact

Overhanging restauration

Ruematoid arthritis

Subgingival calculus

MOST COMMON IN TEETH/BONE

 

Most common impacted tooth: Mn 3, Mx M3, Mx C

Most common congenital missing teeth: M3, MN PM2, MX LI (in primary LI)

Most common occlusion in primary teeth: Edge to Edge

Most likely crowded out of mandibular arch: MN PM 2, from retention of primary Mn M2

Most likely crowded/blocked out of maxillary arch: MX C

Most common reason of maxillary tori removal: Prosthetic treatment (23% and woman)

Most common permanent tooth have caries: MN M1 Least common: Mand incisors

Most common complication of extraction: Root fracture

Caries in radiotherapy patient mostly in: Cervical

 

Most common impacted tooth from the following

1 max canine

2 man 1st premolar

3 max lateral

4 man canine

Ans 1


Most common congenital missing from the following

1 max lateral

2 man canine

3 max premolars

4 max canine

Ans 1

RADIOLOGY

Collimation: Control size & shape of X-ray beam, reduce area of exposure (reduce volume of irradiated tissues), reduce amount of scatter radiation by 60%, Reduce x-ray beam size/diameter. Rectangular preferred * cant be bigger than 75 inches of pt face

Rectangular preferred: (lead)

Filtration(aluminum): selectively absorbs low energy, high wavelength x-ray/energy (total filtration cant be more than 2.5 mm)

Penumbra: the fuzzy, unclear area that surrounds a radiographic image

Larger Penumbra: Decreased CONTRAST, Decreased SHARPNESS

Less Penumbra: Increased CONTRAST, Increased SHARPNESS

Sharpness Influenced by: Focal spot size, Film composition, Movement

Reduce/Prevent Penumbra: Reduce Object-Film distance (X-ray should be parallel to long axis of tooth and as close as possible), decrease Spot-Object distance (use 8 in PID), No MOVEMENT

Best revealing issue for prediction about Ossification: Hand wrist radiograph

sinus/orbital rim: Waters (If waters is not option, then CT scan)

Mid facial fracture: Waters

Zygomatic arch/Zygomatic fracture: SMV (Submentovertex)

Mandibular fracture: Panoramic

Condylar fracture: Reverse Towne

Mandibular symphysis fracture, sialolithiasis in Wharton’s ducts: Occlusal

Mandibular Symphysis Fracture: Antero-posterior or pano

Fracture of Angle, body, and ramus: Lateral oblique

Errors in angulation:

Vertical: x-ray needs to be perpendicular to film & object (if too much angulation= foreshortening. too little angulation=elongation*elong more common) usually 8-10 degrees is needed (5-10 degrees)

Horizontal angulation causes distortion of image: Overlap, superimposition contacts

Central ray not in middle: cone cut

Pano: If chin is down (steeper smile): max anterior teeth elongated & narrow, mandibular anterior teeth appear foreshortened & widen

If chin is to high (frown line): Max anteriors appear frowned foreshortened & widen Mand anteriors elongated & narrow (Reverse smile line/ positive occlusal plane AKA

PID length changed from 16 to 8 inch: beam 4 times intense

PID length changed from 8 to 16 inch: beam 1/4 times intense

Kvp: beam quality, ability for the beam to penetrate tissues, energy (quality & quantity), Only Kvp and Filtration affect Contrast.

mA: A beam radiation quantity, density & patient dose (quantity)

to Increased contract and density in X-ray: increase Kvp, mA, exposure time, reduce obj-film

Max permissible dose of radiation in a year:

Occupational: 5 rem/year
or 0.05sv/year

Dental professional: 5 rem/year, 50 msv/year, 4 msv/month, 0.8 msv/week

Non occlpational/prego assitant: 0.1 rem/year, 0.0001 sv/year

Radio-sensitive: Immature blood cells esp lymphocytes /bone marrow, Reproductive (#1), Intestine, Mucous Membrane

Radio-resistant: Muscles (#1), Nerves, heart, mature bone, salivary glands

First thing that will happen after high dose of radiation? Erythema

Horizontal fracture: Multiple vertical angulated radiographs (PA)

 

MRI which type of radiation

1 electromagnetic radiowaves

2 ionizing radiation (UV, x-ray, gamma ray, light waves)
Ans 1

 

Most readiosensitive to x-ray: Bone, muscle , Nerve or thyroid (mucous membrane)

 

When a radiographic examination is warranted for a 10 year old child, the most effective way to decrease radiation exposure is to

use a thyroid collar and lead apron.
B. apply a radiation protection badge.
C. use high speed film (EKTA F-type is the best)
D. decrease the kilovoltage to 50kVp.
E. take a panoramic film only

Ans C primarily (ways to reduce pt radiation, speed films, lead apron, increase filtration, lead diaphragm in con, rectangular collimator, inc source fim distance, intensifying screens (used in pano and ceph) so if all of the above option, pick that

 

1 pano=2 bitewings

 

Sinus appears inferior to the roots of molars, which radiographic technique used?

A- Periapical parallel
B- Bisecting bitewing

C- Parallel bitewing

D- Periapical bisecting

Ans D

 

A higher kilovoltage produces x-rays with:

A Greater energy levels

B More penetrating ability

C Shorter wavelengths

D Increase in density

E All

Ans: E. All. higher kilovoltage (higher kVp produces higher contrast scale too so more greys)

 

68-Why to wash film by water in the last step in processing

processing: to remove excess chemicals

purpose of processing: gain latent image

developer: turns all exposed areas to black metallic silver, last step restrainer: removing unexposed silver halide crystals to reduce fog

fixing: stops developing.last step hardener: hardens gelatin, shortens drying time, protect film

fixing always twice as long as developing

Ans: 1

 

102 during panorama x ray Patient was moved for 1 sec what will happen

1 horizontal distortion

2 vertical distortions in the inferior border of mandible in this area when he moved

3 blurred all the film

4 no thing
5. the film was reversed (placed backwards) during exposure (will get herringbone effect)
Ans: 3 if patient/cone moves during exposure its blurry, vertical distortion will happen if vertical angulation changes, horizontal distortation if head moves left/right

 

117- if you compare the radiation dose of the person working in a nuclear power plant and that of the dental assistant or whoever takes the x-Rey in the dental office, how much will the dosage of that person be?

1\10 of the nuclear worker

10 times of the nuclear worker

5 times of the nuclear worker

1\5 times of the nuclear worker

Ans A

 

In health, the crest of the alveolar bone, as seen in a radiograph, is situated 1~to~2mm apical to the CEJ. Radiographically, the normal alveolar crest should parallel an imaginary line drawn between the cemento-enamel junction of adjacent teeth.

Both statements are true

What cannot be seen with a PA radiograph?
A. Pterygoid hamulus
B. coronoid notch/mandibular formane
C. mental foramen
D. mandibular Canal
E. coronoid process
Ans B (notch cant be seen, cornoid process can be seen)

Filtration is used in dental x-ray machines to remove

A- Scatter radiation photons

B- High energy electrons

C- Long wavelength photons

D- Low energy electrons

Ans C (Filtration is a mechanism where the low quality, long wavelength X-rays)

 

Crown – root ratio and residual support can best be test:

Bite Wings film

Panoramic

PA film bisecting angle technique (bisecting angle reduces exposure time b/c use of short cone, but distorted)

PA paralleling technique(paralleling technique shows less distortion, increase exposure time b/c long cone used)
ans D

 

 cephalometric doesn’t show:

a.ethmoid sinus

b.sphenoid sinus

c.maxillary sinus

d.frontal sinus

Ans A (use ruler as magnification)

 

Identify Q: ear lobe, M:Coronoid process, O: stylohyoid (s.s)? N: angle, H: gosht image of contralateral mandible. K: dorsum of tongue. F: embrasure spot/black triangle
Hyoid bone zygomatic process. Maxillary sinus, Identify the u shape in x ray: zygomatic process (u above 1st Mx M1)

 

DISEASE IN RADIOLOGY:

 

Soap bubble/step ladder appearance: Odontogenic myxoma

Soap/honey: ameloblastoma

Honey combed appearance: aneurysmal bone cyst

Scoop out radioL: histiocytosis x/langerhans

Cotton wool appearance: Paget’s disease

Scalloped in Mn PM: traumatic bone cyst

Ground glass appearance: Fibrous dysplasia (McCune with map of maine café au lait +hyperPTH+fibrous dyspl)

Sunburst appearance: Osteosarcoma

Swish cheese appearance histology: Adenoid cystic carcinoma

Ghost teeth: regional odontodysplasia

Ghost cells: Calcifying odontogenic cyst/gorlin cyst

Radiolucency like dentigerous w snowflake around Mx C: Adenoid odontogenic tumor AOT

Radiolucency around crown: Dentigerous cyst

Radiolucency with driven snow calcification: CEOT/pindborg

Asymptomatic radiolucencies and transform radiopacities: Cemento-osseous dysplasia

Sequestered bone seen in Xray: Osteomyelitis

Multiple myeloma: punched out radiolucency

 

PEDIATRIC

Dental lamina forms at 6w but teeth begin to calcify 2nd trimester

During initiation: supernumerary (most in Mx anterior mesioense), agenesis (least Mx canine)

During cap stage (proliferation): fusion (in primary, 2 Root, tooth count 1 less), germination (1Root normal tooth count), odontoma, dense in den (permanent MX LI), cyst

during bell histo : AI, DI bell morpho: peg lateral, macrodoncia, taurodontism, concrescene, dens evaginatus

During apposition: enamel hypoplasia, pearls, concresence

During maturation and Calcification/mineralization: Fluorosis(enamel), tetracyclin(dentin)

thickness of coronal dentin in primary teeth, compared with permanent teeth: ½

thickness of coronal enamel in primary teeth, compared with permanent teeth: s.s…

Most common class for pedo: flush terminal (will turn to edge to egde or class I)

If distal terminal: most likely class II

If mesial terminal: most likely (class I or III)
after tx of child, most common: traumatic lip injury

the difference between the dimensions of primary C&D&E and permanent canine+ first and second premolars? leeway space (mandibular 5mm, maxillary 3mm)

the most critical primary tooth to be lost? 2nd molar

which type of growth menarche related to? skeletal or dental or .. cant remember other options .. i picked skeletal s.s???

Most common cause of sealant failure: moisture

Sealant retention: micromechanical

Common reason amalgam fails: inadeqeuate depth
How many permanent teeth does a 9 year old have in the mouth. 9-12

Child LA dose= Child’sweight (lb)/150(lb) x Adult dose.
APF percentage – 1.23%

HHS recommends fl- level to be at 0.7mg/L but EPA: max 4mg/L (usually b/w 0.7-1.2)

Lethal dose fluoride in adult: 2.5-10g (4-5g)

Lethal dose fluoride child: 5 mg/kg (500mg)
Fl community (74%)
If indicated, minimum age to prescribe fluoride: 6months until 16 years

When determining the appropriate dose of systemic fluoride supplement for a child, it is MOST important for the dentist to consider which of the following

The fluoride content of the drinking water

the child’s diet and caries activity

c. the child age and the fluoride content of the drinking water

the child’s weight and the fluoride content of the drinking water.

Ans c

 

What is the optimal average amount of fluoride, in ppm, for public drinking water of most communities?

A- 0.5

B- 0.7

C- 1.0

D- 1.5

Ans B ( 74% of communities are flurodated)

The optimal concentration of fluoride for community, water depends upon:
A) the proportion of residents who are children.

B) temperature of the air.

C) caries rate of the total population.

Ans B cold climate: 1.2 ppm, warmer climate: 0.7 ppm

 

RQ Recent tests have shown that out of the following one is the most effective:

Apf 1.23% for 25 secs

Fluoride mouthwash

Sodium fluoride gel tray for 1 min

Fluoride varnish

Ans D

 

What is the difference between treating an odontogenic infection in children and adult?

Children more bleeding tendency

Children more at risk for leukocytopenia

More likely to dehydrate

 

For incipient caries what do you recommend?

-varnish (25% lowered)

-silver amide fluoride

-composite

Ans A (if sealant there better 70% lowered caries risk)

 

Child has history of generalized growth failure (“failure to thrive”) during first 6 months of life, following dental sequelae:

A Enamel hypoplasia

B retrusive mandible

C retrusive midface

D small permanent teeth

ans A (Crown calcification of permanent teeth begins at birth (first molar) and between 3-

12 months for the anterior teeth.)

129- Most common location for caries in a 4 year old child?

Distal of mandibular m2

Mesial of mandibular m1

Distal of mandibular m1

Distal of mandibular canine

Ans C (distal of primary MN M1 and mesial of MN M2)

 

In attempting to correct a single tooth anterior crossbite with a removable appliance, Which of the following is the most important for the dentist to consider?

A- Making periodic adjustments

B- Incorporating maximum retention

C- Patient’s overbite

D- Making sure there is adequate space

(fixed appliance with finger spring appliance do it ASAP during mixed dentition)

 

Superficial decalcification and staining are evident in the buccal groove of a mandibular molar. Which of the following is the treatment of choice?

Oral prophylaxis at 3-month intervals
b. Daily irrigation with pressurized water

Complete elimination of the area by preparation and restoration

Enameloplasty, limited to the superficial depth of the decalcified enamel

Ans D

 

 permanent tooth bud is accidentally extracted while removing a primary molar,

Immediately placed back on the crypt,

throw it away and inform the parents

others

Ans A

 

What to make sure to do in Pedo anesthesia?

aspirate and administer anesthetic slowely

administer only one cartridge

others

ans A (limit of lido 4.4mg/kg)

 

if you give 1 carpule and kid starts feeling agitates:

Intravenous admin

Allergic to anesthetic

Allergic to epinephrine

ANS a

 

Which of the following accounts for the main cause of failure of replanted teeth?

Ankylosis

Infection

Pulpal necrosis

internal resorption

external resorption or inflamm same

Ans E

 

What determines the level of a class II gingival seat on primary tooth?

1.Based on the Contact of the adjacent tooth

2.axial depth

3.Gingival papillae

4.Marginal gingiva
ans  could be a or 2?

 

 

All of these reasons are why you keep mand 2molar in mouth that’s ankylosed as long as possible except:

A Mesialization of per 1m

B Distalization of pm1

C Supraeruption of opposing tooth

D Maintain bone width

Ans B

 

The late mesial shift of a permanent first molar primarily the result of closure of:
A. Canine

B) Leeway (occurs when you loose E)

C) Primate

Extraction
Ans B (early mesial shift is when permanent M1 erupts because of primate space)

 

45- 4 years child Primary central intruded 5 mm what to do

1 ortho pull

2 surgical extract

3 let re erupt

Ans 3 (if specifically says its touching tooth bud, then EXT according to DD)

A 6 year old patient has an intrusive injury to tooth 5.2 All of the following are possible sequelae to the permanent successor EXCEPT

enamel hypoplasia.

root dilaceration.

delayed eruption.

ectopic eruption

Ans A because by age 6 Cr formation already formed.

 

3 years old comes to have what do you see:

Plaque and pellicle

Nothing

Bacteria

Nasthmiyh

Ans D

 

The permanent first molars of a 7 year old patient have pronounced, deep occlusal fissures that are stained. Bite-wing radiographs show a normal dentino-enamel junction. The most appropriate treatment is

conservative amalgam restorations.
B. glass-ionomer restorations.
C. application of pit and fissure sealants.
D. topical fluoride application.
E. observation
ans C

 

Among the following which is the best material to obturate the primary tooth in pulpectomy?

Caoh2

Zoe

Ans. Primary B ZOE so that it reabsorbs (A if permanent for apixification)

 

132- A radiograph of a 4-year-old child reveals no evidence of calcification of mandibular second premolars. This means that

these teeth may develop later.

the child will probably never develop second premolars.

it is too early in life to make any final predictions concerning the development of any permanent teeth

extraction of primary second molars should be performed to allow the permanent first molars to drift forward.

Ans 1

 

Five hours ago, a 12-year-old boy fell and fractured his maxillary right central incisor at

the level of the gingival tissue. The exposed pulp is vital. For this tooth, tx:

Pulp cap

Extraction

Pulpotomy

D) complete root-canal treatment

Pulpectomy and placement of calcium hydroxide
ans D

 

The success of a pulpotomy for a primary molar depends primarily upon:

A vital root pulp

the patient’s age.

C the amount of root resorption:

the absence of internal root resorption.

 

14 yo has a good oral hygiene, lives in community fluoridated area. For prevention of future caries what should be instructed:

A better oral hygiene

B placing sealants in all 4 1st molars,

C fluoride gel placement by pt daily,

D fluoride rinse daily after brushing.
ans: D (fl- until 16yr most effect, sealant from 6-12.5)

 

14 yo she has a little brown discoloration in her mandibular 1st molar occlusal pit and fissure, with an explorer catching on it, you should:

A place sealant over

B preventive resin restoration

C amalgam,

D no treatment at this time.

Ans: B (if she was 12, I would put A)

 

the MOST crucial element in sealant retention is:

depth of fissures

number of pits

moisture control during placement

type of sealant (light cure vs self cure)

length of curing time

Ans C

 

Which of the following shows 200% of its adult growth by age 9-10 years?

Lymphoid

General

Genital

Neural

Ans A

 

9.5 year old child has a white spot on the facial surface of permanent maxillary central incisor. Condition is due to:

hypocalcificaion secondary to trauma to primary dentition

Hypoplastic defect secondary to systemic infection at 6-12 mnth of age

disturbance during morphodiferentiation stage of tooth development

hypercalcified enamel secondary to increased ca uptake into tooth at 6-12 month of age

ans 1

 

84- 11 years old child with buccal erupted canine what to expected to see

1 gingival Recession

2 anterior deep bite

Ans 1

 

The radiographs of a 9 year old with tooth 1.1 completely erupted and tooth 2.1 unerupted reveal a palatally located mesiodens. The most appropriate management is to

monitor the eruption of

2.1 for another year. B. uncover the mesiodens, wait for eruption and then extract it.

extract the mesiodens and allow passive eruption of 2.1.

extract the mesiodens and orthodontically extrude 2.1

Ans C (remove mesiodense with a palatal flap)

 

to test newly erupted tooth: cold?

 

Case with pulpal necrosis after trauma on completed root formation tooth, treatment to do

Apexification

Apexogenisis

pulpotomy

Pulpectomy(thisone)

 

Same question above but with incomplete root formation? Apexification

 

ORTHODONTICS

 

space for eruption of MN molars is due to: resorption of ant border of ramus

Epiphyseal plates? Synchondrosis

191 which class: class 2 div 1 (me, kan is 2.2)

Temporary anchorage device (mini implants in ortho) can be transosteally, subperiosteally, or endosteally in intraalveolar or extraalveolar. Use: to move teeth then removed after tx complete

Orthodontic uprighting of a mandibular second molar that has tipped mesially into the space of a missing mandibular first molar in an otherwise intact dentition may result in

decreased overbite
B. intrusion of the mandibular second molar
C. extrusion of the mandibular second premolar
D. initial mobility of the mandibular second molar
E. development of posterior crossbite
Ans A

 

What happens if there is premature exfoliation of mandibular primary canine?

need space maintainer

shift of midline to affect side

mandibular incisors move distally and lingually

decrease in arch length

ans. B (tuft) but if ALL of the above, go with that (because needs lingual arch, C is correct from dentin and decrease in arch length too)

 

pt with maxillary arch constriction of 3mm and a posterior cross bite. What will you see?

Normal midline

Midline shift towards the unaffected side

Midline shift toward the affected side
ans. C (midline shifts twds affected side but functional shift away from affected side)

 

Vertical bone loss in ortho T/F

Heavy force, pulling with large movement

Light force, pulling with large movement

Heavy force, pulling with small movement

Ans C

 

What happens with inter-canine distance after mixed dentition

increased

decreased

stable, no change
Ans.C increases during mixed dentition then stabilizes)

 

Orthodontic correction of a maxillary midline diastema is most appropriate in a/an

8 year old patient without a thumb sucking habit.
B. 14 year old patient without a thumb sucking habit.
C. 8 year old patient with a thumb sucking habit.
D. 14 year old patient with a thumb sucking habit
ans B (correct diasthemas only when canines are out)

 

When do you do serial extractions?

For space deficiency in maxilar anterior region

For space deficiency in max posterior region

For space deficiency in mand anterior region

For space deficiency in mandíbular posterior region
And. C (I thnk A and C)

Serial extraction required: For sever arch length descripancy (more than 10mm in Bolton analysis) in mixed dentition. Done in both
Mandibular symphysis fuse/closed: 6-9 months

 

Serial extraction

involves the 4 permanent first premolars. B. is the treatment for Class II skeletal malocclusions with severe space shortage. C. is commenced with bilateral expansion of the arches. D. is best suited to Class I dental and skeletal malocclusions with minimal space shortage. E. requires leeway space maintenance

 

The space for the eruption of permanent mandibular second and third molars is created by the:

apposition of the alveolar process.

apposition at the anterior border of the ramus.
C. resorption at the anterior border of the ramus.

Ans C

 

When a simple tipping force is applied to the crown of a single-rooted tooth, the center of rotation is located

at the apex

at the cervical line

within the apical half of the root

within the cervical one third of the root.

Ans yuri says D (group says C) check

 

The Frankel functional regulator appliance performs all of the following EXCEPT

increasing vertical dimension.
B. repositioning the mandible forward.
C. retraction of the maxillary molars.
D. expansion of the dental arches

ANS A (asked yuri, check learn understand)

 

46- In forced excursion of central incisors u pull it:

1mm in 1 week

2mm in 1 week

1mm in 1 month

2 mm in 1 month

Ans 1

 

76-Which is hardest to maintain space missing primary

1 max 1st molar in 7 years

2 man 1st molar in 6 years

3 man 2nd molar in 5 years

Ans 3

 

109 – scaring from the electrical burn, never treated and it is in the corner of the mouth, what will happen?

arch length discrepancy

impacted tooth

decrease VDO

labial inclination of teeth

Ans C.

 

The advantage of external split over internal?

Rigidity

Durability

Conservative to the tooth

 

Patient wants an implant on space between teeth but not sufficient enough space for implant. Dentist provided coil spring to create space. What type of movement

a) intermittent force

b) continuous force

c) interrupted force

Ans B

 

33 years old Male Fat Patient had septal deviation suffer from sleep apnea, which one of the following is NOT related to sleep apnea

Patient gender

Patient age

obesity

Septal deviation

ans. 2

 

Class III patient: which of the following is not helpful in establishing whether pt has retrognathic maxilla or prognathic mandible?

photographs

study models

ceph analysis

clinical examination

Ans. B (don’t show relation of jaws, just dento alveolar)

 

Which of the following is NOT true regarding orthodontic tooth movement?

A- Blood flow within the PDL is altered after force application

B- Pulpal tissue activates a neural response

C- Chemical changes in the compressed PDL stimulate cellular differentiation

D- Oxygen tension is increased in some areas of the PDL and decreased in other areas.

ANS: B

 

 

An 8-year-old patient has a permanent maxillary first molar extracted because of caries. The best approach to prevent malocclusion is to:

place a space maintainer.

wait for the second molar to erupt and drift mesially into the space.

extract the mandibular first molar to equalize the tooth-size ratio.

extract the contralateral maxillary first molar to maintain arch.symmetry

 

 

 

 

 

 

 

 

 

 

 

 

 

 

185- diagnosis : reticular Lichen planus

186 – diagnosis : Florid cemento osseous dysplasia

187- Diagnosis antral pseudo cyst

 

 

MALIGNANCY of jaw will make:

Tooth displace

Root resorption

Cortical expantion

Cortical destruction

 

Informed consent can have all of the following EXCEPT:
A) Informed consent must

be presented in advance of the treatment.
B) Informed consent must contain treatment options.
C) Informed consent must be in written form.
D) Informed consent must contain risks and benefits of the treatment

 

analysis of a study is in

method

discussion

conclusion

 

DAY 2:

https://www.aapd.org/media/Policies_Guidelines/BP_CariesRiskAssessment.pdf

 

 

cephalometric analysis: to check profile must use soft tissue (point Sb-subnasale and Pog’)

rule is for magnification and ethmoid sinus is only one that cannot be seen (frontal, maxillary and sphenoid are identifiable)

 

For LAP: give metronidazole.
For endo: pen V, amoxi, augmentin, clinda, metro, clarithro, azithro, doxy but lots of resistance (not ceph, not erythro, no Cipro and def NOT mino)

 

Smoking pt: pharma: Buspirone or Chantix

 

If pt is taking antibiotics and needs prophylaxis, we give them from another category

 

In which cases do we treat with conscious? In which with NO2 and in which with tell show do?

 

 

Bad taste of drugs? a)Xanax; b)aspirin c)fosamax d)some beta-blocker

Meds cause Bad taste: Metronidazole, Chantix, Carbamazepine, ACE inhibitors, clarithromycin

 

When to premedicate:

extractions, periodontal procedures. (surgery, SRP, probing, and recall maintenance), implant placements of avulsed teeth, endodontic (RCT) instrumentation of surgery ONLY past the apex, subgingival placement of antibiotic fibers/strips, initial placement of orthodontic bands (not brackets), intraligamentary local anesthetic injections, prophylactic cleaning of teeth or implants where bleeding is anticipated. Sulcular (PDL) injection (because bacteria filled sulcus can cause bacteremia)

 

NOT to premedicate:
Restorative Dentistry: with or without a retraction cord, local anesthetic injections (non- intraligamentary), intra-canal RCT, post and core placement, placing rubber dams, post-operative suture removal, placement of RPD and orthodontic appliances, impressions, fluoride treatments, radiographs, or shedding of primary teeth.

 

ASA

https://www.asahq.org/standards-and-guidelines/asa-physical-status-classification-system

 

Bacteria causing SABE: alpha hemolytic, viridans streptococcal organisms, including Streptococcus mutans, S. mitor, S. salivarius, and S. sanguis.

 

Absolute Implant Contraindications: recent MI or CVA, valvular prosthesis surgery, immunosuppression, hemostatic issues that impair normal healing cascade, active malignancy treatment, drug abuse, psychiatric illness and IV bisphosphonates.

Relative Implant Contraindications: osteoporosis, smoking, uncontrolled diabetes, alcoholism, + interlukin-1 genotype, HIV positive, cardiovascular disease, hypothyroidism, post-irradiated mandible or maxilla, and poor oral hygiene. girls before 15, boys before 18

related categories

Dentistry Exams INBDE Resources



Leave a Reply

Your email address will not be published. Required fields are marked *